Vous êtes sur la page 1sur 102

1) Concernant l’indication de la thrombolyse, Indiquez la proposition exacte

Embolie pulmonaire
Dissection aortique
Péricardite
Ulcère gastrique
Accident vasculaire cérébrale hémorragique
Concernant l’indicat

2) Concernant la contre-indication de la thrombolyse, Indiquez la proposition exacte


Accident vasculaire cérébrale hémorragique
Infarctus du myocarde
Embolie pulmonaire
Thrombose de prothèse valvulaire mécanique
Occlusion artérielle aiguë
Concernant la contre

3) Concernant l’indication de l’héparine, indiquez la proposition exacte


Syndrome coronarien aiguë
Dissection aortique
Péricardite
Ulcère gastrique
Accident vasculaire cérébrale hémorragique récent
Concernant l’indicat

4) Concernant l’effet secondaire indésirable de l’héparine, indiquez la proposition exacte


Hyperleucocytose
Hyperkaliémie
Thrombopénie
Hyponatrémie
Augmentation de la créatininémie
Concernant l’effet s

5) Concernant l’indication de l’anti vitamine K, indiquez la proposition exacte


Maladie thromboembolique veineuse
Déficit de coagulation
HTA non contrôlée
Insuffisance hépatocellulaire
Accident vasculaire cérébrale récent
Concernant l’indicat

6) Concernant la contre-indication de l’anti vitamine K, indiquez la proposition exacte


Insuffisance hépatocelluclaire
Maladie thromboembolique veineuse
Embolie pulmonaire
Fibrillation auriculaire
Infarctus du myocarde avec thrombus dans le ventricule gauche
Concernant la contre

1
1) Un homme de 67 ans, habite à PP, hospitalisé en urgence au Service Porte de l'hôpital Calmètte, le
05/07/14 à 23h30mn pour une dyspnée d'aggravation brutale depuis 1 jour, il vous explique qu'il est
toujours essoufflé de base et plutôt limité dans ses activités quotidiennes mais qu'actuellement il ne
peut plus faire un mouvement sans être très essoufflé. L'interrogatoire retrouve une notion d'infarctus
5 ans auparavant sans notion de stent ainsi qu'une consommation alcoolique 5 à 6 bouteilles de bières
par jour, avec tabagisme actif à 24 PA. Il n'y a pas eu de douleur thoracique. Son poids: 95kg pour
une taille de 165cm2; TA: 130/80; Fc: 108bpm; SaO2: 92%; T: 36,8. L'auscultation cardiaque
retrouve un souffle holosystolique à 2/6 au 4ème EICG en jet de vapeur. On retrouve aussi des
crépitant au 2 bases pulmonaires, des œdèmes des membres inférieures, une turgescence jugulaire et
reflux hépatojugulaire; ECG et la radiographie pulmonaire sont les suivants:
Le critère diagnostique IC est:
Les signes clinique insuffisance respiratiores,
Les signes fonctionnels et singes physiques insuffisance cardiaques avec les preuves de
diminution FEVG en échocardiographie ou en IRM Cardiaque.
Oedème MIs, turgescence jugulaire, hépatomégalie et hépatalgie.
Les syndrome insuffisance hépotocellulaire.
Les preuve de diminution FEVG en échocardiographie ou en IRM
Un homme de 67 ans,

2) Un homme de 67 ans, habite à PP, hospitalisé en urgence au Service Porte de l'hôpital Calmètte, le
05/07/14 à 23h30mn pour une dyspnée d'aggravation brutale depuis 1 jour, il vous explique qu'il est
toujours essoufflé de base et plutôt limité dans ses activités quotidiennes mais qu'actuellement il ne
peut plus faire un mouvement sans être très essoufflé. L'interrogatoire retrouve une notion d'infarctus
5 ans auparavant sans notion de stent ainsi qu'une consommation alcoolique 5 à 6 bouteilles de bières
par jour, avec tabagisme actif à 24 PA. Il n'y a pas eu de douleur thoracique. Son poids: 95kg pour
une taille de 165cm2; TA: 130/80; Fc: 108bpm; SaO2: 92%; T: 36,8. L'auscultation cardiaque
retrouve un souffle holosystolique à 2/6 au 4ème EICG en jet de vapeur. On retrouve aussi des
crépitant au 2 bases pulmonaires, des œdèmes des membres inférieures, une turgescence jugulaire et
reflux hépatojugulaire; ECG et la radiographie pulmonaire sont les suivants:
Consernant la dyspnée de ce patient:
Il s'agit d'une dyspnée NYHA III
Il s'agit d'une dyspnée NYHA IV
Il s'agit d'une dyspnée NYHA IV sur un état de base stade III
Il s'agit d'une dyspnée NYHA IV sur un état de base stade II
Il s'agit d'une dyspnée NYHA II sur un état de base stade I.
Un homme de 67 ans,

3) Un homme de 67 ans, habite à PP, hospitalisé en urgence au Service Porte de l'hôpital Calmètte, le
05/07/14 à 23h30mn pour une dyspnée d'aggravation brutale depuis 1 jour, il vous explique qu'il est
toujours essoufflé de base et plutôt limité dans ses activités quotidiennes mais qu'actuellement il ne
peut plus faire un mouvement sans être très essoufflé. L'interrogatoire retrouve une notion d'infarctus
5 ans auparavant sans notion de stent ainsi qu'une consommation alcoolique 5 à 6 bouteilles de bières
par jour, avec tabagisme actif à 24 PA. Il n'y a pas eu de douleur thoracique. Son poids: 95kg pour
une taille de 165cm2; TA: 130/80; Fc: 108bpm; SaO2: 92%; T: 36,8. L'auscultation cardiaque
retrouve un souffle holosystolique à 2/6 au 4ème EICG en jet de vapeur. On retrouve aussi des
crépitant au 2 bases pulmonaires, des œdèmes des membres inférieures, une turgescence jugulaire et
reflux hépatojugulaire; ECG et la radiographie pulmonaire sont les suivants:
L'ECG met en évidence
Tachycardie sinusal avec fc à 108bpm, QRS fin; HVC et surcharge systolique.
Tachycardie sinusal avec HVG.
BAV II Mobitz I
BAV I

1
Sus-décalage du segment ST en V6
Un homme de 67 ans,

4) Un homme de 67 ans, habite à PP, hospitalisé en urgence au Service Porte de l'hôpital Calmètte, le
05/07/14 à 23h30mn pour une dyspnée d'aggravation brutale depuis 1 jour, il vous explique qu'il est
toujours essoufflé de base et plutôt limité dans ses activités quotidiennes mais qu'actuellement il ne
peut plus faire un mouvement sans être très essoufflé. L'interrogatoire retrouve une notion d'infarctus
5 ans auparavant sans notion de stent ainsi qu'une consommation alcoolique 5 à 6 bouteilles de bières
par jour, avec tabagisme actif à 24 PA. Il n'y a pas eu de douleur thoracique. Son poids: 95kg pour
une taille de 165cm2; TA: 130/80; Fc: 108bpm; SaO2: 92%; T: 36,8. L'auscultation cardiaque
retrouve un souffle holosystolique à 2/6 au 4ème EICG en jet de vapeur. On retrouve aussi des
crépitant au 2 bases pulmonaires, des œdèmes des membres inférieures, une turgescence jugulaire et
reflux hépatojugulaire; ECG et la radiographie pulmonaire sont les suivants:
La radiographie thoracique:
Met en évidence un syndrome de condensation du lobe inférieure gauche
Met en évidence un syndrôme alvéolo-interstitiel bilatéral
N'était pas indiquée dans la prise en charge de ce patient
Met en évidence 2 opacités spiculées du lobe supérieur droit
Met en évidence un index cardio-thoracique supérieure à 0,5.
Un homme de 67 ans,

5) Un homme de 67 ans, habite à PP, hospitalisé en urgence au Service Porte de l'hôpital Calmètte, le
05/07/14 à 23h30mn pour une dyspnée d'aggravation brutale depuis 1 jour, il vous explique qu'il est
toujours essoufflé de base et plutôt limité dans ses activités quotidiennes mais qu'actuellement il ne
peut plus faire un mouvement sans être très essoufflé. L'interrogatoire retrouve une notion d'infarctus
5 ans auparavant sans notion de stent ainsi qu'une consommation alcoolique 5 à 6 bouteilles de bières
par jour, avec tabagisme actif à 24 PA. Il n'y a pas eu de douleur thoracique. Son poids: 95kg pour
une taille de 165cm2; TA: 130/80; Fc: 108bpm; SaO2: 92%; T: 36,8. L'auscultation cardiaque
retrouve un souffle holosystolique à 2/6 au 4ème EICG en jet de vapeur. On retrouve aussi des
crépitant au 2 bases pulmonaires, des œdèmes des membres inférieures, une turgescence jugulaire et
reflux hépatojugulaire; ECG et la radiographie pulmonaire sont les suivants:
L'épisode actuel pourrait correspondre à:
OAP cardiogénique secondaire à un rétrécissement aortique serré
SCA ST+
Pneumopathie franche lobaire aiguë du lobe inférieur gauche
Insuffisance cardiaque aiguë (OAP) sur insuffisance cardiaque globale chronique
Exacerbation aiguë de BPCO.
Un homme de 67 ans,

6) Un homme de 67 ans, habite à PP, hospitalisé en urgence au Service Porte de l'hôpital Calmètte, le
05/07/14 à 23h30mn pour une dyspnée d'aggravation brutale depuis 1 jour, il vous explique qu'il est
toujours essoufflé de base et plutôt limité dans ses activités quotidiennes mais qu'actuellement il ne
peut plus faire un mouvement sans être très essoufflé. L'interrogatoire retrouve une notion d'infarctus
5 ans auparavant sans notion de stent ainsi qu'une consommation alcoolique 5 à 6 bouteilles de bières
par jour, avec tabagisme actif à 24 PA. Il n'y a pas eu de douleur thoracique. Son poids: 95kg pour
une taille de 165cm2; TA: 130/80; Fc: 108bpm; SaO2: 92%; T: 36,8. L'auscultation cardiaque
retrouve un souffle holosystolique à 2/6 au 4ème EICG en jet de vapeur. On retrouve aussi des
crépitant au 2 bases pulmonaires, des œdèmes des membres inférieures, une turgescence jugulaire et
reflux hépatojugulaire; ECG et la radiographie pulmonaire sont les suivants:
Les étiologies suivantes pourraient être évoquées:
Non-observance médicamenteuse
Ecart au régime sans sel
Poussée hypertensive

2
Ischémie myocardique
Valvulopathie aiguë.
Un homme de 67 ans,

7) Un homme de 67 ans, habite à PP, hospitalisé en urgence au Service Porte de l'hôpital Calmètte, le
05/07/14 à 23h30mn pour une dyspnée d'aggravation brutale depuis 1 jour, il vous explique qu'il est
toujours essoufflé de base et plutôt limité dans ses activités quotidiennes mais qu'actuellement il ne
peut plus faire un mouvement sans être très essoufflé. L'interrogatoire retrouve une notion d'infarctus
5 ans auparavant sans notion de stent ainsi qu'une consommation alcoolique 5 à 6 bouteilles de bières
par jour, avec tabagisme actif à 24 PA. Il n'y a pas eu de douleur thoracique. Son poids: 95kg pour
une taille de 165cm2; TA: 130/80; Fc: 108bpm; SaO2: 92%; T: 36,8. L'auscultation cardiaque
retrouve un souffle holosystolique à 2/6 au 4ème EICG en jet de vapeur. On retrouve aussi des
crépitant au 2 bases pulmonaires, des œdèmes des membres inférieures, une turgescence jugulaire et
reflux hépatojugulaire; ECG et la radiographie pulmonaire sont les suivants:
L'échocardiographie retrouve un ventriculaire gauche dilaté (DTDVG:72mm) avec hypokinésie
globale et FEVG = 30% ainsi qu'une insuffisance mitrale modérée sur Valves fine. Le principale
étiologie à évoquer pour expliquer les anomalies du VG est:
Cardiopathie hypertensive.
Cardiomyopathie ischémique
Cardiomyopathie rythmique chronique
Cardiomyopathie des maladies de système
Cardiomyopathie alcoolique.
Un homme de 67 ans,

8) Un homme de 67 ans, habite à PP, hospitalisé en urgence au Service Porte de l'hôpital Calmètte, le
05/07/14 à 23h30mn pour une dyspnée d'aggravation brutale depuis 1 jour, il vous explique qu'il est
toujours essoufflé de base et plutôt limité dans ses activités quotidiennes mais qu'actuellement il ne
peut plus faire un mouvement sans être très essoufflé. L'interrogatoire retrouve une notion d'infarctus
5 ans auparavant sans notion de stent ainsi qu'une consommation alcoolique 5 à 6 bouteilles de bières
par jour, avec tabagisme actif à 24 PA. Il n'y a pas eu de douleur thoracique. Son poids: 95kg pour
une taille de 165cm2; TA: 130/80; Fc: 108bpm; SaO2: 92%; T: 36,8. L'auscultation cardiaque
retrouve un souffle holosystolique à 2/6 au 4ème EICG en jet de vapeur. On retrouve aussi des
crépitant au 2 bases pulmonaires, des œdèmes des membres inférieures, une turgescence jugulaire et
reflux hépatojugulaire; ECG et la radiographie pulmonaire sont les suivants:
Un examen complémentaire indispensable s'impose dès lors pour éclairer la Q7:
ECG
Coronarographie
Echographie abdominale et des veines sus-hépatiques
TDM cardiaque
Scintigraphie myocardique
Un homme de 67 ans,

9) Un homme de 67 ans, habite à PP, hospitalisé en urgence au Service Porte de l'hôpital Calmètte, le
05/07/14 à 23h30mn pour une dyspnée d'aggravation brutale depuis 1 jour, il vous explique qu'il est
toujours essoufflé de base et plutôt limité dans ses activités quotidiennes mais qu'actuellement il ne
peut plus faire un mouvement sans être très essoufflé. L'interrogatoire retrouve une notion d'infarctus
5 ans auparavant sans notion de stent ainsi qu'une consommation alcoolique 5 à 6 bouteilles de bières
par jour, avec tabagisme actif à 24 PA. Il n'y a pas eu de douleur thoracique. Son poids: 95kg pour
une taille de 165cm2; TA: 130/80; Fc: 108bpm; SaO2: 92%; T: 36,8. L'auscultation cardiaque
retrouve un souffle holosystolique à 2/6 au 4ème EICG en jet de vapeur. On retrouve aussi des
crépitant au 2 bases pulmonaires, des œdèmes des membres inférieures, une turgescence jugulaire et
reflux hépatojugulaire; ECG et la radiographie pulmonaire sont les suivants:
Le souffle évoqué dans l'énoncé est en rapport avec:

3
Rétrécissement aortique calcifié
Insuffisance aortique
Insuffisance mitrale fonctionnelle par dilatation du VG
Rétrécissement mitral congénital
Insuffisance mitrale sur dégénérescence fibro-élastique
Un homme de 67 ans,

10) Un homme de 67 ans, habite à PP, hospitalisé en urgence au Service Porte de l'hôpital Calmètte,
le 05/07/14 à 23h30mn pour une dyspnée d'aggravation brutale depuis 1 jour, il vous explique qu'il
est toujours essoufflé de base et plutôt limité dans ses activités quotidiennes mais qu'actuellement il
ne peut plus faire un mouvement sans être très essoufflé. L'interrogatoire retrouve une notion
d'infarctus 5 ans auparavant sans notion de stent ainsi qu'une consommation alcoolique 5 à 6
bouteilles de bières par jour, avec tabagisme actif à 24 PA. Il n'y a pas eu de douleur thoracique. Son
poids: 95kg pour une taille de 165cm2; TA: 130/80; Fc: 108bpm; SaO2: 92%; T: 36,8. L'auscultation
cardiaque retrouve un souffle holosystolique à 2/6 au 4ème EICG en jet de vapeur. On retrouve aussi
des crépitant au 2 bases pulmonaires, des œdèmes des membres inférieures, une turgescence
jugulaire et reflux hépatojugulaire; ECG et la radiographie pulmonaire sont les suivants:
Les traitements suivants peuvent être indiqués dans le traitement de l'insuffiance Cardiaque
systolique de stade III:
Durétique si signes congestifs, IEC + Bétabloquant de l'insuffisance cardiaque à dose
minimale efficace,
Diurétique si signes congestifs,
ARA II + anti-aldostérone
Réadaptation cardiaque
Stimulation biventriculaire si FE inférieure à 35% et QRS supérieur à 120ms.
Un homme de 67 ans,

4
1) Les symptômes d'une hypoglycémie insulinique peuvent être masqués par la prise de :
Glucorticoïdes
Inhibiteurs de l'enzyme de conversion
Bêta-bloquants
Diurétiques thiazidiques
Progestatifs macro-dosés
Les symptômes d'une

2) Parmi les symptômes suivants, l’un n’est pas évocateur d’hypoglycémie aiguë. Lequel ?
Signe de Babinski
Coma convulsif
Transpirations profuses
Dyspnée de Kussmaul
Cétonurie faible (+)
Parmi les symptômes

3) Un coma acidocétosique chez un diabétique insulino-dépendant peut être déclenché par :


Un arrêt de l'insuline
Un exercice physique intense
Un surdosage en insuline
Un repas riche en glucide
À jeune prolongé
Un coma acidocétosiq

4) Dans le coma hyperosmolaire du diabétique :


La natrémie est basse
Il existe une acidose métabolique décompensée
La déshydratation est globale
Il y a toujours de cétose
La calcémie est base
Dans le coma hyperos

5) Concernant les comas métaboliques du diabète :


Le coma céto-acidosique s’installe généralement en quelques minutes
Dans le coma hyperosmolaire la déshydratation est à prédominance extra-cellulaire
Dans le coma céto-acidosique, la kaliémie a tendance à augmenter sous l’influence de
l’insulinothérapie
Dans le coma céto-acidosique, la décroissance de la fréquence respiratoire est grossièrement
corrélée à celle de l’acidose
Dans le coma hyperosmolaire, le pH sanguin est toujours normal
Concernant les comas

1
1) M. K TH âgé de 65 ans, vient vous voir en consultation pour la douleur lombaire très intense avec
un déficit sensitive et motrice des 2 membres inférieurs, une constipation depuis 7 jours. Il est en
cours d’un traitement hormonal pour son cancer de la prostate découverte depuis 7 mois. Quel est
votre diagnostic ?
Syndrome de compression médullaire
Calcul rénal.
Effets indésirables de traitement hormonal.
Ostéophyte.
Pyélonéphrite Aiguë.
M. K TH âgé de 65 an

2) M. K TH âgé de 65 ans, vient vous voir en consultation pour la douleur lombaire très intense avec
un déficit sensitive et motrice des 2 membres inférieurs, une constipation depuis 7 jours. Il est en
cours d’un traitement hormonal pour son cancer de la prostate découverte depuis 7 mois. En cas d’un
syndrome de compression médullaire par une métastase osseuse suit à un IRM rachidienne, En cas
de la présence d’un signe de l’épidurite, voulez-vous ajouter quel médicament de plus ?
Paracétamol.
Aspirine.
Corticoïdes.
Ibuprofène.
Diazépam.
M. K TH âgé de 65 an

3) M. K TH âgé de 65 ans, vient vous voir en consultation pour la douleur lombaire très intense avec
un déficit sensitive et motrice des 2 membres inférieurs, une constipation depuis 7 jours. Il est en
cours d’un traitement hormonal pour son cancer de la prostate découverte depuis 7 mois. Dans quel
cas la radiothérapie de la colonne rachidienne est indiquée dans le syndrome de compression
médullaire compliqué par un cancer de la prostate ?
Le traitement chirurgical est possible.
Après l’intervention chirurgicale.
L’apparition du déficit neurologique dans les 48 premières heures.
L’apparition du déficit neurologique dans les 24 premières heures.
L’apparition du déficit neurologique dans les 72 premières heures.
M. K TH âgé de 65 an

4) M. K TH âgé de 65 ans, vient vous voir en consultation pour la douleur lombaire très intense avec
un déficit sensitive et motrice des 2 membres inférieurs, une constipation depuis 7 jours. Il est en
cours d’un traitement hormonal pour son cancer de la prostate découverte depuis 7 mois. Dans
certain cancer, un syndrome de lyse tumorale apparait
2 semaines après le traitement par la chimiothérapie.
Quelque mois plus tard après avoir finis le traitement.
Le plus souvent pendant la chimiothérapie ou dans les jours qui suivent (entre 1 et 5 jours).
Lorsqu’il y a des sites métastatiques multiples.
3 semaines après le traitement par la chimiothérapie.
M. K TH âgé de 65 an

5) M. K TH âgé de 65 ans, vient vous voir en consultation pour la douleur lombaire très intense avec
un déficit sensitive et motrice des 2 membres inférieurs, une constipation depuis 7 jours. Il est en
cours d’un traitement hormonal pour son cancer de la prostate découverte depuis 7 mois. Qu’est-ce
qu’on voit à la radiographie thoracique dans un syndrome cave supérieur compliqué par une tumeur?
Opacité micronodulaire.

1
Encombrement des sinus costo diaphragmatique.
Elargissement du médiastin.
Epaississement bronchique.
Opacité retrouvé surtout dans l’apex du poumon droite.
M. K TH âgé de 65 an

6) M. K TH âgé de 65 ans, vient vous voir en consultation pour la douleur lombaire très intense avec
un déficit sensitive et motrice des 2 membres inférieurs, une constipation depuis 7 jours. Il est en
cours d’un traitement hormonal pour son cancer de la prostate découverte depuis 7 mois. Comment
traiter une thrombose diagnostiquée chez un patient cancéreux ?
Commencer tout d’abord pas Aspirine.
Héparine Bas Poids Moléculaire 200 UI/kg/j (en 1 ou 2 injections SC).
Puis Anticoagulant orale SINTROM avec INR > 2,0 et < 3,0.
Héparine Bas Poids Moléculaire 200 UI/kg/j jusqu’à une amélioration.
Héparine Bas Poids Moléculaire 400 UI/kg/j jusqu’à une amélioration.
M. K TH âgé de 65 an

7) M. K TH âgé de 65 ans, vient vous voir en consultation pour la douleur lombaire très intense avec
un déficit sensitive et motrice des 2 membres inférieurs, une constipation depuis 7 jours. Il est en
cours d’un traitement hormonal pour son cancer de la prostate découverte depuis 7 mois. Concernant
le critère de la gravité de la neutropénie fébrile, le taux des Polynucléaires neutrophiles (≤ 500 - <
1000/mm3) est classé dans quel grade de l’OMS ?
Normal
Grade 1
Grade 2
Grade 3
Grade 4
M. K TH âgé de 65 an

8) M. K TH âgé de 65 ans, vient vous voir en consultation pour la douleur lombaire très intense avec
un déficit sensitive et motrice des 2 membres inférieurs, une constipation depuis 7 jours. Il est en
cours d’un traitement hormonal pour son cancer de la prostate découverte depuis 7 mois. Quel est la
première question urgente pourra être posé au patient dans le cas d’une suspicion de la neutropénie
fébrile suite à une chimiothérapie d’un cancer?
En cas de fièvre : Le malade est-il neutropénique
Vous avez quoi comme le type de cancer
Quelle est le protocole de la chimiothérapie que vous avez reçu
Quand est-ce que vous avez reçu le traitement
Est-ce que c’est la première cure de traitement
M. K TH âgé de 65 an

9) M. K TH âgé de 65 ans, vient vous voir en consultation pour la douleur lombaire très intense avec
un déficit sensitive et motrice des 2 membres inférieurs, une constipation depuis 7 jours. Il est en
cours d’un traitement hormonal pour son cancer de la prostate découverte depuis 7 mois.
Quel est le risque principal causer le décès dans la neutropénie fébrile suite à une chimiothérapie
d’un cancer?
La déshydratation causée par la fièvre.
Choc cardiogénique.
Choc septique.
Choc hypovolémique.
Choc anaphylactique.
M. K TH âgé de 65 an

2
10) M. K TH âgé de 65 ans, vient vous voir en consultation pour la douleur lombaire très intense
avec un déficit sensitive et motrice des 2 membres inférieurs, une constipation depuis 7 jours. Il est
en cours d’un traitement hormonal pour son cancer de la prostate découverte depuis 7 mois. Le
recours au Facteurs de croissance hématopoïétiques de manière préventive, prophylaxie secondaire,
est indiqué dans quel cas ?
Dès la première cure da la chimiothérapie.
Suite à un épisode de neutropénie fébrile post chimiothérapie.
Avant commencer la chimiothérapie.
S’il y a une demande d’un patient.
Patient à haut risque d’une infection grave.
M. K TH âgé de 65 an

11) S DR âgé de 75 ans, vient vous voir en consultation pour la douleur lombaire très intense avec un
déficit sensitive et motrice des 2 membres inférieurs, une constipation depuis 7 jours. Il est en cours
d’un traitement hormonal pour son cancer de la prostate découverte depuis 7 mois.Quel est votre
diagnostic ?
Syndrome de compression médullaire
Calcul rénal.
Effets indésirables de traitement hormonal.
Ostéophyte.
Pyélonéphrite Aiguë.
S DR âgé de 75 ans,

12) En cas d’un syndrome de compression médullaire par une métastase osseuse suit à un IRM
rachidienne, En cas de la présence d’un signe de l’épidurite, voulez-vous ajouter quel médicament de
plus ?
Paracétamol.
Aspirine.
Corticoïdes.
Ibuprofène.
Diazépam.
En cas d’un syndrome

13) Dans quel cas la radiothérapie de la colonne rachidienne est indiquée dans le syndrome de
compression médullaire compliqué par un cancer de la prostate ?
Le traitement chirurgical est possible.
Après l’intervention chirurgicale.
L’apparition du déficit neurologique dans les 48 premières heures.
L’apparition du déficit neurologique dans les 24 premières heures.
L’apparition du déficit neurologique dans les 72 premières heures.
Dans quel cas la rad

14) Dans certain cancer, un syndrome de lyse tumorale apparait


2 semaines après le traitement par la chimiothérapie.
Quelque mois plus tard après avoir finis le traitement.
Le plus souvent pendant la chimiothérapie ou dans les jours qui suivent (entre 1 et 5 jours).
Lorsqu’il y a des sites métastatiques multiples.
3 semaines après le traitement par la chimiothérapie.
Dans certain cancer,

15) Qu’est-ce qu’on voit à la radiographie thoracique dans un syndrome cave supérieur compliqué

3
par une tumeur?
Opacité micronodulaire.
Encombrement des sinus costo diaphragmatique.
Elargissement du médiastin.
Epaississement bronchique.
Opacité retrouvé surtout dans l’apex du poumon droite.
Qu’est-ce qu’on voit

16) Comment traiter une thrombose diagnostiquée chez un patient cancéreux ?


Commencer tout d’abord pas Aspirine.
Héparine Bas Poids Moléculaire 200 UI/kg/j (en 1 ou 2 injections SC).Puis Anticoagulant
orale SINTROM avec INR > 2,0 et < 3,0.
Anticoagulant orale SINTROM avec INR > 2,0 et < 3,0 puis Héparine Bas Poids
Moléculaire 200 UI/kg/j (en 1 ou 2 injections SC).
Héparine Bas Poids Moléculaire 200 UI/kg/j jusqu’à une amélioration.
Héparine Bas Poids Moléculaire 400 UI/kg/j jusqu’à une amélioration.
Comment traiter une

17) Concernant le critère de la gravité de la neutropénie fébrile, le taux des Polynucléaires


neutrophiles (≤ 500 - < 1000/mm3) est classé dans quel grade de l’OMS ?
Normal
Grade 1
Grade 2
Grade 3
Grade 4
Concernant le critèr

18) Quel est la première question urgente pourra être posé au patient dans le cas d’une suspicion de
la neutropénie fébrile suite à une chimiothérapie d’un cancer?
En cas de neutropénie grade 3 ou 4 : Le malade est-il fébrile En cas de fièvre : Le malade
est-il neutropénique
Vous avez quoi comme le type de cancer
Quelle est le protocole de la chimiothérapie que vous avez reçu
Quand est-ce que vous avez reçu le traitement
Est-ce que c’est la première cure de traitement
Quel est la première

19) Quel est le risque principal causer le décès dans la neutropénie fébrile suite à une chimiothérapie
d’un cancer?
La déshydratation causée par la fièvre.
Choc cardiogénique.
Choc septique.
Choc hypovolémique.
Choc anaphylactique.
Quel est le risque p

20) Le recours au Facteurs de croissance hématopoïétiques de manière préventive, prophylaxie


secondaire, est indiqué dans quel cas ?
Dès la première cure da la chimiothérapie.
Suite à un épisode de neutropénie fébrile post chimiothérapie.
Avant commencer la chimiothérapie.

4
S’il y a une demande d’un patient.
Patient à haut risque d’une infection grave.
Le recours au Facteu

5
1) Quelle est la proposition exacte concernant l’épidémiologie de cancer chez l’enfant ?
. L’enfant n’est jamais atteint de cancer
. Il n’existe pas de cancer chez l’enfant au Cambodge
. Il ne faut jamais penser au cancer devant une masse abdominale de nouveau-née
. L’incidence de cancer de l’enfant < 15ans est environ 130 nouveaux cas par million par an
. La leucémie aigue est une maladie de l’adolescence
Quelle est la propos

2) Lequel entre les cancers si dessous est un cancer chez l’enfant ?


. Le carcinome hépatocellulaire
. Le cholangiocarcinome
. Le carcinome à petite cellule du poumon
. Le carcinome épidermoide de l’œsophage
. Le néphroblastome
Lequel entre les can

3) Quelle est la localisation tumorale de neuroblastome ?


. Le rein
. La glande surrénale et le system nerveux sympatique
. Le ganglion lymphatique
. La moelle osseuse
. La rétine
Quelle est la locali

4) La différence entre le cancer chez l’enfant et chez l’adulte :


. Pas de différence
. Le développement de cancer chez l’adulte est plus vite
. Les enfants répondent mieux au traitement
. Les cellules cancéreuses chez l’adulte sont plus tôt immatures
. Les localisations et les types histologiques sont très semblables
La différence entre

5) La conduit à tenir de cancer chez l’enfant:


. Evoquer le diagnostique, préserver la fonction vitale, traitement de la douleur, organiser la
prise en charge par un équipe spécialisé
. Le traitement est dans le bute palliatif
. La chimiothérapie doit être commencée immédiatement
. La radiothérapie ne doit pas pratiquer chez l’enfant
. La chirurgie doit être faite avant la chimiothérapie
La conduit à tenir d

1
1) Évaluation pré-thérapeutique essentielle ( Pre-therapeutic assessment )concernant l’infection à
virus C traitée par DAA ( Direct Acting Antiviral) + Ribavirin quelle est la proposition qui n’est pas
importante ?
- Génotypage du virus
- Evaluation du stade de la fibrose
- Dosage de la charge virale ARN VHC ( RNA HCV )
- Dosage IL 28 .B Génotype
- L’utilisation du contraceptif efficace
Évaluation pré-théra

2) Parmi les propositions suivantes concernant la surveillance du malade de l’hépatite C après l’arrêt
du traitement , laquelle est exacte pour confirmer la guérison ?
- Dosage de la charge virale PCR au 2ème mois après l’arrêt du traitement
- Dosage de la charge virale PCR au 3ème mois après l’arrêt du traitement
- Dosage de la charge virale PCR au 6 ème et 12ème mois après l’arrêt du traitement
- Dosage de la charge virale PCR au 7ème mois après l’arrêt du traitement
- Dosage de la charge virale PCR au 16 ème mois après l’arrêt du traitement
Parmi les propositio

3) Donner la réponse exacte concernant l’utilisation de SOFOSBUVIR + LEDIPASVIR.Parmi Les


effets secondaires souvent rencontrés :
- Anurie
- Anémie
- Amaigrissement
- Impuissance sexuelle
- Céphalée et Fatigue
Donner la réponse

4) Donner la réponse exacte concernant l’utilisation de SOFOSBUVIR + LEDIPASVIR.Ces


médicaments sont efficaces :
- Pour tout les genotypes 1 2 3 4 5 6
- Pour le genotype 2- 3 seulement
- Pour le genotype 1-2- 3 seulement
- Pour le genotype 1 – 4- 5- 6 seulement
- Pour le genotype 3- 4 seulement
Donner la réponse

5) Parmi les propositions suivantes concernant le traitement du malade de l’hépatite C , laquelle est
fausse ? Eradication du virus peut :
- Réduire voire regresser le stade de fibrose,
- Réduire le risque vers le Cancer
- Eviter l’évolution vers le cancer
- Réduire les besoins de transplantation hépatique
– Reduire le risque de décompensation
Parmi les propositio

6) Parmi les propositions suivantes concernant la surveillance et le résultat à la fin de traitement du


malade de l’hépatite C , laquelle est fausse ?
. Le Dosage de la charge virale s’effectue au 3ème - 6ème et 12 ème mois après l’arrêt du
traitement

1
. Le contrôle de la charge virale est nécessaire à chaque année
. Si la charge virale s’avère négative au 12 ème mois on peut considerer comme guérit
. Le contrôle de la charge virale n’est pas nécessaire si elle est negative au 12 ème mois
après l’arrêt du traitement
- L’eradication du virus peut réduire voire regresser le stade de fibrose,
Parmi les propositio

7) Quel est le médicament en Cardiologie qu’il faut éviter d’utiliser en association avec
SOFOSBUVIR + LEDIPASVIR :
. Propranolol
. Atenolol
. Amiodarone
. Nifédipine
. Enalapril
Quel est le médicame

8) Parmi les medicaments hypolipidémiants cités ci dessous, Quel est le médicament qui est sans
interaction avec SOFOSBUVIR + LEDIPASVIR :
. Atorvastatin
. Lovastatin
. Gemfibrozil
. Fenofibrate
. Simvastatin
Parmi les medicament

9) Traitement de l’hépatite virale B : parmis les médicaments cités en bas lequel est préférable et
nouveau dans l’année 2017 ?
. Lamivudine
. Adéfovir
. Telbivudine
. Ténofovir disoproxyl fumarate
. Ténofovir Alafénamide
Traitement de l’hépa

10) Traitement de l’hépatite virale B par Ténofovir disoproxyl fumarate doit être en precaution chez
les malades avec :
. Insuffisance rénale
. Insuffisance hépatique
. Diabète
. HTA
. Anémie Hb < 7g
Traitement de l’hépa

11) Traitement de l’hépatite virale B : En cas de résistance avec Tenofovir disoproxyl fumarate faut
il choisir quel medicament :
. Lamivudine
. Adéfovir
. Telbivudine
. Entécavir
. Ténofovir Alafénamide
Traitement de l’hépa

2
12) Traitement de l’hépatite virale B : En cas de résistance avec l’Entécavir faut il choisir quel
médicament:
. Lamivudine
. Adéfovir
. Telbivudine
. Ténofovir disoproxyl fumarate
. Ténofovir Alafénamide
Traitement de l’hépa

13) La prevention de la contamination périnatale de la mère porteuse de l’Ag HBs se fait par :
- Vaccination
- Vaccination et serum HBIG dans 24 h après accouchement
- Vaccination et serum HBIG dans 12 h après accouchement
- serum HBIG dans 24 h après accouchement
- serum HBIG dans 12 h après accouchement
La prevention de la

14) Une femme enceinte de 5 mois avec Ag HBs + et ADN HBV 150.000.000 copies/ml ;
- Prescrire Telbivudine
- Prescrire Ténofovir au 6 ou 7 ème mois de gestation et arrêter au semaine 24 après
l’accouchement
- Prescrire Ténofovir au 6 ou 7 ème mois de gestation et arrêter au semaine 12 après
l’accouchement
- Vaccination et serum HBIG de la mère au 6 ème mois
- Prescrire Ténofovir au 6 ou 7 ème mois de gestation et arrêter au semaine 12 après
l’accouchement + Vaccination et serum HBIG dans 12 h après l’accouchement
Une femme enceinte

15) Une femme HBV enceinte de 4 mois sous traitement avec Lamivudine :
. Continuer le traitement
. Arrêter le traitement
. Changer à l’entécavir
. Associer avec Ténofovir
. Changer au Ténofovir
Une femme HBV encei

16) Une femme porteuse de l’Ag HBs en accouchement :


. Vaccination et serum HBIG pour la mère
. Vaccination et serum HBIG pour le nouveau né dans 24 h après l’accouchement
. Prescrire Ténofovir pour la mère
. Vaccination et serum HBIG pour le nouveau né dans 12 h après l’accouchement
. Vaccination et serum HBIG pour le nouveau né dans 12 h après l’accouchement et
Prescrire Ténofovir pour la mère
Une femme porteuse d

3
1) Quelle est la valeur de la clairance de la créatinine en dessous de laquelle on parle d’insuffisance
rénale chronique terminale ?
40ml/mn
30ml/mn
20ml/mn
10ml/mn
5ml/mn
Quelle est la valeur

2) Qu’est ce qu’une fistule artério-veineuse ?


Anastomose artério-veineuse
Anastomose veino- veineuse
Anastomose veino lymphatique
Pontage aorto-fémorale
anastomoses porto-cave
Qu’est ce qu’une fis

3) Parmi les propositions suivantes concernant le bain de dialyse, Choisissez une fausse reponse :
Liquide stérile
Contient du sodium, du potassium et du calcium
Contient du glucose, de l’actate et de l’eau
Dépourvu des solutés dont l’élimination est désirée
Composition proche du plasma
Parmi les propositio

4) Où se situe l’extrémité interne du cathéter de dialyse péritonénale ?


Dans l’intestin grêle
Dans le colon
Dans le cul de sac de Douglas
Dans la cavité pleurale
Dans la vessie
Où se situe l’extrém

5) Quelle est l’endroit anatomique classique d’une fistule artério-veineuse ?


Face antérieur de l’avant bras
Face postérieur de l’avant bras
Face antérieur de la cuisse
Face postérieur de la cuisse
Face interne de la cuisse
Quelle est l’endroit

6) Quel est le temps moyen pour obtenir une fistule artério-veineuse fonctionnelle ( cicatrisation,
veine artérialisée)?
2 semaines
3 semaines
Entre 4 à 5 semaines
Entre 5 à 6 semaines
.A partir de 6ème semaine
Quel est le temps mo

1
7) Expliquer ce qu’est le phénomène de diffusion au cours de l’hémodialyse :
Transfert d’eau à la faveur d’un gradient de pression (de la plus forte vers la plus faible).
Transfert des petites molécules à travers des PORES de la membranes semi-perméable. Ce
transfert se fait en fonction de la taille des molécules et du gradient de concentration de part et
d’autre de la membrane (de la plus forte concentration vers la plus faible).
Transfert des grandes molécules à travers des pores de la membranes semi-perméable.
Transfert des petites molécules à travers des PORES de la membranes semi-perméable. Ce
transfert se fait en fonction de la taille des molécules et du gradient de concentration de part et
d’autre de la membrane (de la plus faible concentration vers la plus forte).
Transfert d’eau à la faveur d’un gradient de pression (de la plus faible vers la plus forte).
Expliquer ce qu’est

8) Parmis les complications d’une séance d’hémodialyse suivantes, Choisissez une fausse réponse :
Crampes
Hyperthermie
Hypotension artérielle
Sd pseudobulbaire
Embolie gazeuse
Parmis les complicat

9) Quel est le nombre moyen hebdomadaire de séance(s) d’hémodialyse ?


1
2
3
4
7 (1/j)
Quel est le nombre

10) Quelle est la durée moyenne d’une séance d’hémodialyse ?


3 à 4h
4 à 5h
4 à 6h
6 à 8h
4 à 8h
Quelle est la durée

11) Les indications de l’hémodialyse en urgence, sauf :


Hyperkaliémie sévère chez IRCT
OAP chez IRC
Hyperphosphatémie chez IRCT
Sd urémique chez IRCT
Acidose sévère chez IRCT
Les indications de l

12) Néphropathie Glomérulaire :Quel est l’examen préférentiel parmi les suivants, qui permet
d’analyser la glomérulopathie ?
Scanographie rénale
Urographie intraveineuse
Ponction biopsie rénale
IRM rénale
Electrophorèse des protéines urinaires

2
Néphropathie Gloméru

13) Parmis les signes de syndrome néphrotique suivants, Choisissez une fausse réponse:
Hypoalbuminémie
Hypoprotidémie
Albuminurie > 3g/j
HTA
Oedème généralisé
Parmis les signes de

14) Parmis les propositions de syndrome néphrotique pur, Choisissez une fausse réponse :
Hypoalbuminémie
Hypocalcémie
Anomalie de la coagulation
Hématurie macroscopique
Baisse des gammaglobulines
Parmis les propositi

15) Quelle est la classe thérapeutique d’antihypertenseurs qui a démontré ses effets positifs sur la
réduction de la protéinurie chez un patient ayant un syndrome néphrotique (même en l’absence
d’HTA associée)?
Diurétique de l’anse
Thiazidique
Inhibiteurs calciques
Inhibiteurs de l’enzyme de conversion
Antagonistes de l’angiotensine II
Quelle est la classe

16) Les signes de glomérulonéphrite, Sauf :


Hématurie (macroscopique ou microscopique)
HTA
Ictère franche
Protéinurie
Oedème et Insuffisance rénale
Les signes de glomér

17) En cas de Syndrome néphrotique avec oedèmes importants, quel conseil donner sur les apports
hydriques et en chlorure de sodium (NaCL)?
Apport hydrique est strictement interdit, Apports en NaCL: entre 2 à4g/j
Apport hydrique : restreints (limités à la soif), Apports en NaCL: entre 2 à4g/j
Apport hydrique : restreints (limités à la soif), Apports en NaCL: entre 5 à6g/j
Apport hydrique est strictement interdit, Apports en NaCL: entre 1 à3g/j
Apport hydrique : restreints (limités à la soif), Apports en NaCL: entre 6 à10g/j
En cas de Syndrome n

18) Je suis utile en cas de poussée oedèmateuse chez un patient ayant un syndrome néphrotique avec
insuffisance rénale aigue. J’ai une action pharmacologique intense mais brève. Je suis entre autre un
antihypertenseur. QUI SUIS JE ?
Diurétique proximale
Diurétique thiazidique
Diurétique de l’anse

3
Diurétique du tube collecteur cortical
Substance osmotique
Je suis utile en cas

19) Quel est le temps moyen d’apparition de la glomérulonéphrite aigue post streptococcique après
l’infection (angine à streptocoque non traitée)?
Immédiat
2 jours
Entre 2 et 5 jours
Entre 4 et 15 jours
Entre 4 et 21 jours
Quel est le temps mo

20) Je suis responsable de la glomérulonéphrite aigue post streptococcique. Je suis une bactérie, mais
quell est mon nom ?
Streptocoque .A
Streptococcus pneumoniae
streptocoque de groupe .B
Streptocoque .B
Streptocoque .D
Je suis responsable

21) Quelle est la tranche d’âge préférentielle touchée par la maladie de Berger?
5-15ans
10-20ans
15-35ans
25-45ans
55-65ans
Quelle est la tranch

22) Principes de traitement de glomérulonéphrite aigue post streptococcique, Chosissez une fausse
réponse :
Repos au lit
Mise sous corticoide
Restriction sodée
Apport hydrique se fera en fonction de la diurèse
Restriction protéique si la créatinine et l’urée sont élevées.
Principes de traitem

23) Trouble de l’érection : Le centre réflexe de l’érection est situé au niveau de :


L’hypothalamus
Le lobe temporal
La moelle thoracique
La moelle lombaire
La moelle sacrée
Trouble de l’érectio

24) Les signes cliniques qui permettent d’évoquer une impuissance sexuelle d’origine vasculaire
chez un homme jeune comportent :
Anéjaculation
Instabilité de l’érection et Abolition des pouls fémoraux

4
Ejaculation précoce et Asthénie intense
Disparition de la libido et perte d’appetit
Abolition des pouls radiaux
Les signes cliniques

25) Les examens complémentaires dans le bilan de trouble de l’érection, sauf un(e), la ou lequel?
PSA
Bilan hormonal
Echodeppler pénien
Scanner abdomino-pelvien
Rigidimétrie
Les examens compléme

26) Les étiologies de dysfonctions érectiles, sauf un(e), la/lequel?


Neurologique
Vaculaire
Endocriniennes
Psychogènes
Cardiaque
Les étiologies de d

27) Définition de PRIAPISME :


Erection prolongé > 3h et en dehors de stimulation sexuelle
Erection prolongé > 5h et hors de stimulation sexuelle
Erection prolongé > 3h et hors de stimulation sexuelle
Erection prolongé > 10h et en dehors de stimulation sexuelle
Erection prolongé > 1h et en dehors de stimulation sexuelle
Définition de PRIAPI

28) La contre indication de SILDENAFIL :


Hypertention artérielle pulmonaire
Hypotention au repos
Insuffisance Rénale
Insuffisance hépatique
Insuffisance veineuse
La contre indication

5
1) La calcitonine chez l’homme:
. Provoque une hypercalcémie
. Provoque une hyperphosphorémie
. Agit essentiellement la formation de la vitamine .D
. Bloque la résorption osseuse ostéoclastique
. Constitue un traitement efficace des hypocalcémies
La calcitonine chez

2) Un syndrome d'hypercalcémie peut s'accompagner d'un des plusieurs signes suivants:


. Polydypsie
. Prurit
. Diarrhée
. Polyphagie
. Trouble mentruel
Un syndrome d'hyperc

3) Laquelle est la cause d’une hypercalcémie à PTH augmenté:


. Intoxication par la vitamine .D
. Une hyperthyroïdie
. Hémopathies malignes
. Un adénome parathyroïdien
. Syndrome des buveurs de lait
Laquelle est la caus

4) Parmi les causes d'hypercalcémie suivantes, quelle est celle comportant une hyperabsorption
digestive du calcium?
. Maladies inflammatoires
. Cancer osseux métastatique
. Hypervitaminose .D
. Immobilisation prolongée
. Maladie de Basedow
Parmi les causes d'h

5) Parmi les propositions suivantes, lequel est trouvé dans l’hypercalcémie?


. Signe de Trousseau
. Allongement de l’espace QT
. Hyperreflexie
. Constipation
. Insomnie
Parmi les propositio

6) La calciurie est abaissée par:


. L'hypercalcémie
. L'acidose métabolique
. Le traitement par le furosémide
. L’hyperhydratation salée
. Les diurétiques thiazidiques
La calciurie est aba

7) Parmi les circonstances suivantes, laquelle peut révéler une hyper-parathyroidie primitive ?

1
. Insuffisance rénale chronique
. Hypercalcémie de découverte fortuite
. Fracture osseuse
. Hypertension artérielle
. Polyarthrite rhumatoïde
Parmi les circonstan

8) Au cours d'une hypercalcémie maligne sans insuffisance rénale, laquelle est la premiàre mesure
thérapeutique à mettre en place en urgence?
. Diurétiques de l'anse (furosémide)
. Réhydratation salée isotonique IV
. Prednisone
. Digitaline
. Diphosphonates
Au cours d'une hyper

9) Pour traiter une hypercalcémie, vous pouvez prescrire un des médicaments suivants:
. Diurétique du groupe thiazide
. Diurétique de l'anse (furosémide)
. Anti-inflammatoires non stéroïde
. Anti-thyroïdiens de synthèse
. Les vitamines D3 actives
Pour traiter une hyp

10) Au cours d'une hypercalcémie, les traitements suivants peuvent être employés sauf un, lequel?
. Diurétiques de l'anse (furosémide)
. Réhydratation
. Prednisone
. Biphosphonate
. Indométhacine
Au cours d'une hyper

11) Vous préconisez à traiter de l’hypercalcémie par la glucocorticoïde car:


. Elle inhibe l’action de l’enzyme au niveau hépatique
. Elle inhibe l’action de l’enzyme au niveau rénale
. Elle entraine une hypersécrétion de PTH 1,84
. Elle provoque une diurèse rénale abondante
. Elle inhibe l’absorption intestinale du calcium
Vous préconisez à tr

12) Parmi les proposition suivantes, lequel est trouvé dans l’hypercalcémie aiguë majeure?
. Une fièvre élevée
. Un calcule renal
. Un signe de trousseau
. Une hyperphosphatémie
. Une diarrhée profuse
Parmi les propositio

13) Les signes biologiques de l’hypercalcémie sont les suivants, sauf:


. Hypercalcémie
. Hypercalciurie

2
. Hypocalciurie
. PTH 1,84 élevée
. Hypophosphatémie
Les signes biologiqu

14) Laquelle est la cause d’une hypercalcémie à PTH diminuée:


. Intoxication par la vitamine.D
. Une hyperparathyroïdie primaire
. Une hyperparathyroïdie secondaire
. Une hyperparathyroïdie tertiaire
. Une hyperplasie parathyroïdienne
Laquelle est la caus

15) Hyperparathyroïdie secondaire peut être causée par une des propositions suivantes, sauf:
. Insuffisance rénale chronique
. Insuffisance en vitamine .D
. Dialyse péritonéale répétée
. Transfusion sanguine répétée
. Insuffisance hépatocellulaire
Hyperparathyroïdie s

16) Hypercalcémie par excès un vitamine .D est traitée par les moyens suivants, sauf:
. Arrêt le supplement en vitamin .D
. Réduction de l’apport de calcium
. Arrêt la prise des thiazidiques
. Réhydratation suffisante
. Prescrire les glucocorticoïdes
Hypercalcémie par ex

3
1) Regarding acute pulmonary edema and cardiogenic shock, which of the following statement is the
most appropriate?
Acute pulmonary edema and cardiogenic shock are a medical emergency
Acute pulmonary edema and cardiogenic shock are treated at home
Acute pulmonary edema and cardiogenic shock are not life-threatening condition
The most common etiology for both is pericarditis
The both are diagnosed by ECG
Regarding acute pulm

2) Which of the following physical examination findings suggests acute pulmonary edema?
Lung crackles + S3 gallop
Pedal edema
Diminished heart sound
Slitting B2
Pericardial friction
Which of the followi

3) Mrs. X-67 years old with a long history of HTN and diabetes mellitus admits to emergency
department with dyspnea at rest preceded by chest pain. Her BP is 70/40mmHg with pulse of
110bpm and SaO2: 88% in room air. Physical examination finds crackles in both lungs with cold
skin.ECG shows Q wave with T negative in leads DII, III and avF. Chest X-ray finds alveolar edema
with left pleural effusion.Which of the following is the most likely diagnosis?
Cardiogenic shock caused by myocardial infarction
Hemorrhagic shock
Bronchopneumonia
Septic shock
Pleural effusion
Mrs. X-67 years old

4) The acute pulmonary edema is suspected, which of the following test is the most appropriate in
diagnosis?
ECG and thoracic TDM
BNP, chest X-ray and echocardiography
CRP and abdominal echography
Cerebral TDM
Hemogram and CRP
The acute pulmonary

5) What is the urgent treatment in acute pulmonary edema with systolic BP > 140mmHg?
Beta blocker
Diuretic and nitrate derivatives
Antibiotic
Inotropic agents
Angiotensin converting enzyme inhibitor
What is the urgent t

1
1) Parmi les signes suivants, laquelle peut typiquement appartenir au tableau d’intoxication aux
benzodiazépines
Une somnolence
Une mydriase unilatérale aréflexique
Une respiration de Cheyne-Stokes
Une asymétrie de tonus
Un allongement du QT sur l’ECG
Parmi les signes sui

2) Parmi les signes cliniques suivants d’une intoxication aux benzodiazépines, laquelle est exacte ?
Une dépression respiratoire
Une mydriase unilatérale aréflexique
Un trouble visuel
Un insomnie
Une hypertension artérielle
Parmi les signes cli

3) Concernant l’intoxication par benzodiazépine, quelle est la réponse exacte ?


Une tachycardie ventriculaire est une complication fréquente
Une arythmie complète par fibrillation auriculaire est une complication fréquente
Un bloc sino-auriculaire de haut degré est une complication fréquente
Une Bradycardie sinusale(Rohypnol®)
Un bloc auriculoventriculaire de haut degré est une complication fréquente
Concernant l’intoxic

4) Parmi les signes cliniques suivants d’une intoxication aux benzodiazépines, laquelle est exacte ?
Une hypertension artérielle
Un trouble visuel
Une mydriase unilatérale aréflexique
Un insomnie
Une hypotension artérielle
Parmi les signes cli

5) Parmi les doses toxicologiques suivantes d’une intoxication aux benzodiazépines en moyenne
chez l’adulte, laquelle est exacte ?
50mg à 70mg
100mg à 500mg
70mg à 90mg
1mg à 5mg
5mg à 10mg
Parmi les doses toxi

6) Parmi les doses toxicologiques suivantes d’une intoxication aux benzodiazépines en moyenne
chez l’enfant, laquelle est exacte ?
50mg à 70mg
100mg à 200mg
70mg à 90mg
1mg à 5mg
5mg à 10mg
Parmi les doses toxi

1
7) Quel est l’antidote d’une intoxication exclusive par benzodiazépines ?
Un flumazénil (Anexate)
Une naloxone
Un N-acétylcystéine
Une vitamine .K
Une vitamine .C
Quel est l’antidote

8) En cas d’intoxication médicamenteuse volontaire aux benzodiazépines, le traitement pré


hospitalisé, laquelle est vraie ?
Une installation en position latérale de sécurité (PLS)
Une pose d’une sonde urinaire
Une antitoxine
Un antidote
Une antibiothérapie
En cas d’intoxicatio

9) En cas d’intoxication médicamenteuse volontaire aux benzodiazépines, le traitement pré


hospitalisé, laquelle est vraie ?
Une installation en position demi assise
Une pose d’une sonde urinaire
Une oxygénothérapie : au masque 8l/mn
Un antidote
Une antibiothérapie
En cas d’intoxicatio

10) En cas d’intoxication médicamenteuse volontaire aux benzodiazépines, le traitement évacuateur,


lequel est vrai ?
Une installation en position demi assise
Une pose d’une voie veineuse avec perfusion d’un soluté glucosé
Un lavage gastrique charbon activé
Un antidote
Une pose d’une sonde urinaire
En cas d’intoxicatio

11) Concernant l’intoxication par les antidépresseurs tricycliques, quelle est la réponse inexacte ?
Un état de mal convulsive
Une tachycardie
Une constipation
Une mydriase unilatérale
Une hypotension artérielle
Concernant l’intoxic

12) Concernant l’intoxication par les antidépresseurs tricycliques, quelle est la réponse exacte ?
Une manifestation allergique cutanéo-muqueuse
Une tachycardie sinusale
Un trouble urinaire
Un trouble génital
Une diarrhée
Concernant l’intoxic

2
13) Parmi les signes cliniques suivants d’une intoxication aux antidépresseurs tricycliques, lequel est
exact ?
Un syndrome anticholinergique
Une mydriase unilatérale aréflexique
Une mydriase unilatérale réflexique
Une bradycardie fréquente
Une pollakiurie
Parmi les signes cli

14) Concernant l’intoxication par les antidépresseurs tricycliques, quelle est la réponse inexacte ?
Une hypertension artérielle
Une tachycardie
Une constipation
Un état de mal convulsive
Une hypotension artérielle
Concernant l’intoxic

15) Parmi les signes cliniques suivants d’une intoxication aux antidépresseurs tricycliques, laquelle
est exacte ?
Une bradycardie fréquente
Une mydriase unilatérale aréflexique
Une mydriase unilatérale réflexique
Une rétention d’urine
Une pollakiurie
Parmi les signes cli

16) Concernant l’intoxication par les antidépresseurs tricycliques, quelle est la réponse inexacte ?
Une tachycardie
Une hypersécrétion bronchique
Une hypotension artérielle
Un état de mal convulsive
Une constipation
Concernant l’intoxic

17) Parmi les traitements suivants d’une intoxication par les antidépresseurs tricycliques, laquelle est
inexacte ?
Une perfusion de lactate de sodium molaire
Une ventilation mécanique
Une oxygenothérapie
Une hémodialyse
Une utilisation du charbon activé
Parmi les traitement

18) En cas d’une intoxication par les antidépresseurs tricycliques, le traitement pré hospitalisé,
laquelle est vraie ?
Une pose d’une voie veineuse
Une pose d’une sonde urinaire
Un antalgique
Une antibiothérapie
Un antidote

3
En cas d’une intoxic

19) Parmi les traitements suivants d’une intoxication par les antidépresseurs tricycliques, laquelle est
inexacte ?
Une lavage gastrique
Une pose d’une sonde vésicale
Une oxygenothérapie
Une ventilation mécanique
Une utilisation du charbon activé
Parmi les traitement

20) En cas d’une intoxication par les antidépresseurs tricycliques, le traitement des torsades de
pointe, laquelle est vraie ?
Une perfusion de soluté glucosé à 10%
Un antidote
Un antalgique
Une pose d’une sonde urinaire
Une utilisation d’isoprotérénol-Isuprel®.
En cas d’une intoxic

4
1) (CS) Une jeunne fille ayant de VIH> 5ans avec CD4 : 45/mm3, et encours de traitement par des
TB et ARV (anti-rétroviral débuté 2 semaines après les Anti- TB) avec une bonne réponde aux
traitements. Mais au J35, elle se présent : Fièvre, asthénie et toux. Quel l`examen nécessaire pour
diagnostiquer PARADOXAL syndrome ?
□Hémoculture
□Examen de crachat contrôle
□Tx CD4 et charge viral
□Radio pulmonaire
□Autre examens pour chercher des infections
(CS) Une jeunne fill

2) (CS) A 19-year-old man has donated blood for the first time. Despite having no risk factors for
human immunodeficiency virus (HIV) infection, his blood tests positive for HIV by enzyme
immunoassay (EIA). Which of the following statements is correct?
□ . EIA is currently the most specific test for HIV
□. He might have a false-positive secondary to an unsuspected collagenvascular disease
□. He has a 75% chance of truly being infected with HIV
□. EIA is an excellent screening test
□. .A Western blot test would be more Sensitive
(CS) A 19-year-old m

3) (CS). A24-year-old woman complains of pain every time she eats or drinks anything. She is HIV
positive, but currently not on any antiretroviral therapy. Her last CD4 count was 250/mm3. Select the
most likely pathogen causing infection.
□ Staphylococcus aureus
□ Candida
□ .P carinii
□ Streptococcus pneumoniae
□ Gram-negative enteric bacilli
(CS). A24-year-old w

4) (CM).A 44-years patient HIV + develops severe cough and shortness of breath on exertion. On
examination, he appears dyspneic, respirations 24/min, pulse 110/min, and oxygen saturation 88%.
His lungs are clear on auscultation and heart sounds are normal. CXR shows bilateral diffuse peri-
hilar infiltrates. Bronchoscopy and bronchial brushings show clusters of cysts that stain with
methenamine silver (silver staining). Which of the following is the most appropriate next step in
management?
□ amphotericin .B
□ cephalosporins
□ trimethoprim-sulfamethoxazole
□ aminoglycosides
□ High O2
(CM).A 44-years pati

5) (CS). A34-year-old man HIV +/under HAART, presents with diarrhea 3 weeks after returning from a trip to kg SOM.
Over the past few weaks, he has gradually developed severe head ache, and lower abdominal pain and diarrhea. Now the
symptoms are much worse with malaise, and a severe head ache, confusion. . He is afebrile, the abdomen is tender in left
lower quadrant, and his stools tests show tinea solium.Which offollowing the initial
□ Cerebral MRI

1
□ Cerebral scanner with radio contrass
□ Lombar puncture
□ Serology test of T. solium antibody
□ Abdominal ultrasound
(CS). A34-year-old m

6) (CS).A31-year-old woman presents with symptoms of vulvar itching and burning made worse by urinating. She has
no fever or frequency, but has noticed a recent whitish vaginal discharge. Clinical examination reveals vulvar erythema,
edema, and fissures. On speculum examination, there is a white discharge with small white plaques loosely adherent to
the vaginal wall. Which of the following treatments is appropriate for her asymptomatic HIV+ male sexual partner?
□ azole cream to the penis
□ oral fluconazole
□ standard urethritis investigation
□ no investigation or treatment
□ azithromycin plus cefixime
(CS).A31-year-old wo

7) (CS). A22-year-old woman HIV+ complains of vulvar itching, burning, and pain when voiding urine. She has no other
symptoms of fever, vaginal discharge, or urinary frequency. Physical examination reveals some vulvar ulceration but no
vaginal discharge. The ulcers are small 2–3-mm lesions with an erythematous base. Which of the following is the most
likely diagnosis?
□ HSV infection
□ Trichomonas vaginalis infection
□ .N gonorrhoeae infection
□ .C. trachomatis infection
□ .M. genitalium infection
(CS). A22-year-old w

8) (CS).A 19-year-old woman HIV+ , CD4 = 450/mm3 was traveling in a rural Kandal. She returned 3 weeks ago and,
over the past few days, has gradually developed lower abdominal pain and diarrhea. Now the symptoms are much worse
with 10 stools a day consisting mostly of mucus and blood. She is febrile, the abdomen is tender in left lower quadrant,
and the remaining examination is normal. Her stool is mostly comprised of blood and mucus. Which of the following is
the most likely causative organism?
□ Escherichia coli infection
□ Salmonella infection
□ Shigella infection
□ Vibrio parahaemolyticus infection
□ histolytica infection
(CS).A 19-year-old w

9) (CS) A 22-year man HIV+ under HAART with viral load of HIV underdetected is back travel from Kampong som.
Ten days after arriving, she develops symptoms of anorexia, malaise, and abdominal cramps followed by a sudden onset
of watery diarrhea. There are no symptoms of fever or chills, and the stools are nonbloody.The most likely pathogen

2
causing infection.
□ Campylobacter
□ .E. coli
□ Salmonella
□ Shigella
□ rotavirus
(CS) A 22-year man H

10) (C S).A 25-year-old HIV+ patient known more than 10 years without Anti Retro Viral treatment . On day he
develops severe head ache and right hemiplesia. On examination, he appears malaise, head ache and right hemiplesia,
BP: 14/8 mm/Hg, pulse 80/min, and his CD4 count 57 cells /mm3 Which of the following is the most appropriate next
step in diagnosis?
□ amphotericin .B
□ test serology of anti body of toxoplasmosis
□ Cerebral scan
□ Lumbar puncture
□ cotrimoxazole
(C S).A 25-year-old

11) (CS) Une jeunne fille ayant de VIH> 5ans avec CD4 : 45/mm3, et encours de traitement par des TB et ARV (anti-
rétroviral débuté 2 semaines après les Anti- TB) avec une bonne réponde aux traitements : CD4 =250/mm3, Charge Viral
HIV : indétectable . Mais au J45, elle se présent : Fièvre, asthénie, céphalée et toux. Que pensez- vous dans cette
situation?
. Nouveau infection opportuniste
. Echec du traitement par des anti-TB
. Réaction paradoxale
. Pneumocystose infection
. Méningite à cryptocoque
(CS) Une jeunne fill

12) (CS) A 19-year-old man has donated blood for the first time. Despite having no risk factors for human
immunodeficiency virus (HIV) infection, his blood tests positive for HIV by enzyme immunoassay (EIA). Which of the
following statements is correct?
. EIA is currently the most specific test for HIV
. He might have a false-positive secondary to an unsuspected collagenvascular disease
.He has a 75% chance of truly being infected with HIV
. EIA is an excellent screening test
.A Western blot test would be more Sensitive
(CS) A 19-year-old m

13) (CS) A 22-year-old married woman has an High risk factors for human immunodeficiency virus (HIV) infection,
( her husband died of HIV+ 1 years ago) her blood tests for many times was Negative for HIV by enzyme immunoassay
(EIA) . Which of the following statements is correct?

3
. EIA test is not so sensitive
. She might have a false-Negative
.She has a 50% chance of truly being non infected with HIV
. She might have a completed CCR5 mutation
.A Western blot test would be more Sensitive
(CS) A 22-year-old m

14) (CS) Parmi les manifestation suivantes, le quelle sont classante pour le stade de SIDA?
. Cachexie
. Carcinoma insitu
. Cancer du sein
. Cancer anal
.Lymphome de Hodgkin
(CS) Parmi les manif

15) (CS) La positivité de l `Antigen p 24 est possible à partir de :


. 7-8jours après infection
.10jours après infection
.12 jours après infection
. 20Jours après infection
.Tout est faux
(CS) La positivité d

16) (CS) la fenêtre virologique de détection virus est de (temps pendant lequel de détection du virus est impossible)
. 7-8 jours
. 10 jours
. 12-15 jours
. 20 jours
. Tout est faux
(CS) la fenêtre viro

17) La prévention de la pneumocystose pulmonaire chez un VIH


. CD4 > 300/mm3
. CD4 < 200/mm3
. CD4 > 250/mm3
. CD4 < 300 > 200/mm3
. tout est faux
La prévention de la

18) Le risque de transmission du VIH d`une mèreà son enfanten absent de traitement :
. Est d`environ 60-65%
. Est d`environ 50-55%
. Est d`environ 40-45%

4
. Est d`environ 30-35%
. Est d`environ 20-35%
Le risque de transmi

19) Parmi les propositions suivantes, lequel est correct :


. Au Cambodge le nombre de personnes infectés par le VIH est estimé à 300000.
. L`incident du VIH en Cambodge est de 20000 par an
. L`incident du VIH en Cambodge est de 10000 par an
. La prévalence du VIH est estimé 0.6 (de population de 15 ans-49 ans) en 2012.
. tout est faux
Parmi les propositio

20) Parmi les propositions suivantes lequel est correcte ?


. Au Cambodge, le nombre de personne infectées par le VIH est estimé à 200000.
. L`incident du VIH au Cambodge est de 60000 par an
. La prévalence du VIH au Cambodge est de 0.9%.
. La prévalence du VIH au Cambodge est de 0.6%.
. Tout est faux.
Parmi les propositio

21) Au Cambodge le diagnostic de VIH est posé formellement sur :


. La positivité de deux Tests Western Blot.
. La positivité de test ELISA+ test rapide
. La positivité de deux tests rapide .
. La positivité de test ELISA+ test Western Blot
. Tout est faux.
Au Cambodge le diagn

22) La positivité d`un test ELISA est possible à partir de :


. 7 jours après infection
. 12-15 jours après infection
. 10 jours après infection
. 20 jours après infection
. 30jours après infection
La positivité d`un t

23) La fenêtre virologique de détection du virus VIH est de :


. 7-8 Jours
. 10 Jours
. 10-15 Jours
. 20 Jours
. 30 Jours
La fenêtre virologiq

5
24) En ce qui concerne le délai d`introduction des antirétroviraux lors d`une infection opportuniste : (S) : semaine
. En cas de cryptococcose méningée, le traitement ARV droit être retardé de 4S si ttt par amphotéricine B.
. En cas de cryptococcose méningée, le traitement ARV droit être retardé de 8S
. En cas de tuberculose, le traitement ARV droit être retardé de 2 semaines après le début du traitement
antituberculeux.
. En cas de tuberculose méningée, le traitement ARV droit être retardé de 2S.
. tout est faux
En ce qui concerne l

25) Quelles infections opportunistes sont plus rencontrées au Cambodge : (CM)


. infection à CMV.
. Infection à cryptococcose.
. Cryptosporidiose.
. infection à mycobacterium avium
. Infection à mycobacterium tuberculose, Infection à cryptococcose.
Quelles infections o

26) Le risqué de transmission du VIH d`une mère à son enfant en l`absent de traitement :
. Est d`environs60-65%
. Est d`environs 50-55%.
. Est d`environs 40-50%
. Est d`environs 30-45%
. Est d`environs 20-35%
Le risqué de transmi

27) Le diagnostic de la méningite à cryptococcose ? :


. La positivité de la sérologie de l`anticorp anti cryptococcose.
. La positivité de la sérologie de l`antigène anti cryptococcose.
. La positivité de test de l`encre de Chine (India ink) , La positivité de la sérologie de l`antigène anti
cryptococcose.
. La positivité de la PCR sur LCR
. Tout est vrai
Le diagnostic de la

28) Quelles sont les antigènes spécifiques du virus VIH-1 ? : (CM)


. Antigène p242
. Ag gp 160
. Ag gp 120
. Ag gp 41, Ag gp 120
. Tout est vrai
Quelles sont les ant

29) (CS). A24-year-old woman complains of pain every time she eats or drinks anything. She is HIV positive, but

6
currently not on any antiretroviral therapy. Her last CD4 count was 250/mm3. Select the most likely pathogen causing
infection.
. Staphylococcus aureus
. Candida
. .P. carinii
. Streptococcus pneumoniae
. Gram-negative enteric bacilli
(CS). A24-year-old w

30) (CM).A 44-years patient HIV + develops severe cough and shortness of breath on exertion. On examination, he
appears dyspneic, respirations24/min, pulse 110/min, and oxygen saturation 88%. His lungs are clear on auscultation and
heart sounds are normal. CXR shows bilateral diffuse peri-hilar infiltrates. Bronchoscopy and bronchial brushings show
clusters of cysts that stain with methenamine silver (silver staining). Which of the following is the most appropriate next
step in management?
.amphotericin .B
. cephalosporins
. trimethoprim-sulfamethoxazole
.aminoglycosides
. High O2
(CM).A 44-years pati

31) (CS). A34-year-old man HIV +/under HAART, presents with diarrhea 3 weeks after returning from a trip to kg
SOM. Over the past few weaks, he has gradually developed severe head ache, and lower abdominal pain and diarrhea.
Now the symptoms are much worse with malaise, and a severe head ache, confusion. . He is afebrile, the abdomen is
tender in left lower quadrant, and his stools tests show tinea solium.
Which of following the initial?
. Cerebral MRI
. Cerebral scanner with radio contrass
. Lombar puncture
. Serology test of T. solium antibody
. Abdominal ultrasound
(CS). A34-year-old m

32) (CS).A31-year-old woman presents with symptoms of vulvar itching and burning made worse by urinating. She has
no fever or frequency, but has noticed a recent whitish vaginal discharge. Clinical examination reveals vulvar erythema,
edema, and fissures. On speculum examination, there is a white discharge with small white plaques loosely adherent to
the vaginal wall. Which of the following treatments is appropriate for her asymptomatic HIV+ male sexual partner?
. azole cream to the penis
. oral fluconazole
. standard urethritis investigation
. no investigation or treatment
. azithromycin plus cefixime
(CS).A31-year-old wo

7
33) (CS). A22-year-old woman HIV+ complains of vulvar itching, burning, and pain when voiding urine. She has no
other symptoms of fever, vaginal discharge, or urinary frequency. Physical examination reveals some vulvar ulceration
but no vaginal discharge. The ulcers are small 2–3mm lesions with an erythematous base. Which of the following is the
most likely diagnosis?
. HSV infection
. Trichomonas vaginalis infection
. .N. gonorrhoeae infection
. .C. trachomatis infection
. .M. genitalium infection
(CS). A22-year-old w

34) (CS).A 19-year-old woman HIV+ , CD4 = 450/mm3 was traveling in a rural Kandal. She returned 3 weeks ago and,
over the past few days, has gradually developed lower abdominal pain and diarrhea. Now the symptoms are much worse
with 10 stools a day consisting mostly of mucus and blood. She is febrile, the abdomen is tender in left lower quadrant,
and the remaining examination is normal. Her stool is mostly comprised of blood and mucus.
Which of the following is the most likely causative organism?
. Escherichia coli infection
. Salmonella infection
. Shigella infection
. Vibrio parahaemolyticus infection
. histolytica infection
(CS).A 19-year-old w

35) (CS) A 22-year man HIV+ under HAART with viral load of HIV underdetected is back travel from Kampong Som.
Ten days after arriving, she develops symptoms of anorexia, malaise, and abdominal cramps followed by a sudden onset
of watery diarrhea. There are no symptoms of fever or chills, and the stools are non bloody. The most likely pathogen
causing infection?
. Campylobacter
. E. coli
. Salmonella
. Shigella
. rotavirus
(CS) A 22-year man H

36) (C S).A 25-year-old HIV+ patient known more than 10 years without Anti Retro Viral treatment . On day he
develops severe head ache and right hemiplesia. On examination, he appears malaise, head ache and right hemiplesia,
BP: 14/8 mm/Hg, pulse 80/min, and his CD4 count 57 cells /mm3 .
Which of the following is the most appropriate next step in diagnosis?
. Amphotericin .B
. Test serology of anti body of toxoplasmosis
. Cerebral scan
. Lumbar puncture

8
. Cotrimoxazole
(C S).A 25-year-old

37) A 27-year-old HIV positive factory worker was brought to the infectious disease department with a 2-week history of
headache, fever, and altered mental status. A lumbar puncture with clear CSF and an increasing intracranial opening
pressure up to 47 cmH2O. What is the most likely diagnosis with this finding?
. Cryptococcocal meningitis
. Bacterial meningitis
. Histoplasmosis
. Tuberculous meningitis
A 27-year-old HIV po

38) A 45-year-old HIV positive man was brought to the infectious disease department with a week history of headache,
fever, and Coma GCS 10/15. A lumbar puncture with turbid CSF and an increasing intracranial opening pressure up to
25 cmH2O. What is your decision?
. Waiting CSF result
. Antibiotic
. Anti TB drugs
. Antifungal drugs
. Amphotericine .B
A 45-year-old HIV po

39) A 45-year-old man HIV positive known more than 5 years without treatment was brought to the infectious disease
department with a three-week history of headache, fever, and Coma GCS 10/15. A lumbar puncture with yellowish CSF
and an increasing intracranial opening pressure up to 30 cmH2O and total lymphocytes count 600 cells/mm3. What is
your decision?
. Waiting CSF result
. Antibiotic
. Amphotericine .B
. Antifungal drugs
. None of above
A 45-year-old man HI

40) A 40-year-old HIV positive woman was brought to the infectious disease department with a one week of severe
headache, fever, and altered mental status. A lumbar puncture with CSF clear an intracranial opening pressure is 35
cmH2O. He was treated as Cryptococcal meningitis and 5 days after she completely improved. One week CSF analysis
showed Cyto-biochemistry is normal. No organisms were observed on Gram stain of the CSF. Negative of India Ink test
and Culture of the CSF also negative.
What is your decision?
. Continue the same treatment
. Discontinue treatment
. Re perform Lumbar Puncture
. CT scan or MRI of brain

9
. Do the Viral load of HIV
A 40-year-old HIV po

41) A 40-year-old HIV positive woman was brought to the infectious disease department with a one week of severe
headache, fever, altered mental and focal neurological signs. After completed patient examination, you think that the first
diagnosis is Cryptococcal neuro meningitis but the patients not allow you to do the LP.
What is your further tests?
. Blood culture
. Serum antigen cryptococcosis
. Serum anti-body cryptococcosis
. MRI of brain
. Scan with contras injection
A 40-year-old HIV po

10
1) Les médicaments suivants sont indiqués dans le traitement de la syphilis, lequel doit-on utiliser
dans la syphilis congénitale:
Pénicilline .G
Extencilline
Doxycline
Erythromycine
Azithromycine
Les médicaments suiv

2) Concernant le protocole thérapeutique de la syphilis dans ses formes tardives (latente plus d’un an
et syphilis III ), montrer la fausse proposition:
Neurosyphilis ou syphilis oculaire: pénicilline G en hospitalisation 0.5 MUl/kg/j en 6
PIV/4h pendant 14 jours
Sans atteinte neurologique centrale: en hospitalisation, au début Extencilline 2.4 MUI/sem. à
j1- j8 et j15 [1.2 MUI chaque fesse]
Syphilis de la femme enceinte: Extencilline 2.4MUI en IM à j1 et j8.
Syphilis congénitale: pénicilline G 100 000 à 150 000 U/kg/j IV à administrer à 50000
UI/kg/12h dans la 1ère semaine de vie, puis toutes les 8h pdt 10 à 14 jours
Aucune
Concernant le protoc

3) Les médicaments suivants sont indiqués dans le traitement du chancre mou, sauf:
Erythromycine
Doxycline
Ciprofloxacine
Ceftriaxone
Azythromycine
Les médicaments suiv

4) Dans le traitement de la gonococcie, lequel des médicaments suivants est prescrit pour le
traitement de l’urétrite (concomitente) à chlamydia:
Ceftriaxone
Spectinomycine
Ciprofloxacine
Doxycline
Aucun ci-dessus
Dans le traitement d

5) Concernant le protocole thérapeutique de la gonococcie dans ses formes compliquées, montrer la


fausse proposition:
Orchi-épididymite: traitement antigonococcique, puis doxycline pendant une semaine
Conjonctivites gonococciques: ceftriaxone 1g IM dose unique
Salpingite: Ceftriaxone IM 500 mg + doxycline et métronidazole jusqu’à apyrexie 2 jours,
puis relais par doxycline et métronidazole 15 jours
Septicémie gonococcique: Ceftriaxone IM 1g jusqu’à apyrexie + 2j, puis relais par
ciprofloxacine ou céfixime po pendant 7 jours
Aucune
Concernant le protoc

6) Laquelle de ces présentations de Econazole est utilisée dans le traitement de la candidose génitale
chez l’homme:

1
Econozole [Gyno-Paveryl 150 mg]
Econazole [Gyno-Paveryl LP]
Econazole [Paveryl lait]
Econazole [Paveryl 1% crème]
Laquelle de ces prés

7) Concernant le traitement des condylomes, quel est le choix pour des formes cutanées vulvaires,
périnéales péri-anales:
Imiquinol 5% crème (Aldara*)
Cryothérapie : application d’azote liquide
Podophylline 25 %
5 fluoro-uracile (Efudix* crème)
Electrocautérisation
Concernant le traite

8) Concernant le traitement des condylomes, quel est votre choix dans les formes diffuses
multifocales et récidivantes.
Imiquinol 5% crème [Aldara*]
Cryothérapie
Podophylline
5 fluoro-uracile [Efudix* crème]
Electrocautérisation
Concernant le traite

9) Concernant le traitement des condylomes, quel est votre choix dans les formes diffuses vulvaires
chez la femm
Imiquinol 5% crème [Aldara*]
Cryothérapie
Podophylline
5 fluoro-uracile [Affudix* crème]
Laser CO2
Concernant le traite

10) Quelle est la proposition thérapeutique qui convient au traitement de l’herpes génital dans la
phase érosive : ZELITREX II : ACICLOVIR creme III : VIOLET DE GENTIANE
I
II
III
I+II
I+III
Quelle est la propos

11) Concernant la conduite à tenir devant les infections sexuellement transmissibles, lequel des
symptomes suivants doit être prescrit par la Doxycycline :
écoulement purulent
écoulement séreux
écoulement cailleboté
écoulement avec mauvaise odeur
écoulement verdatre
Concernant la condui

2
12) Concernant la conduite à tenir devant les infections sexuellement transmissibles, lequel des
symtomes suivants doit être prescrit par le métronidazole :
écoulement purulent
écoulement séreux
écoulement cailleboté
écoulement avec mauvise odeur
Aucun
Concernant la condui

13) Concernant le conseil au patient présentant une insuffisance surrénale, laquelle des propositions
suivantes est la moins importante :
l’hormonothérapie substitutive [hydrocortisone] à instituer à vie
l'adaptation des doses d’hydrocortisone en cas d’agression (infection, chirurgie, stresse…)
le régime est rigoureusement normosodé, les laxatifs et diurétiques à manier avec prudence,
voire à proscrire
La carte d’Addisonien doit indiquer le diagnostic, la nature et la posologie du traitement
prescrit
Le malade doit avoir avec lui de quoi faire réaliser une injection de cortisone stockée à 4º .C
Concernant le consei

14) Concernant la récupération fonctionnelle, l’insuffisance surrénale est réversible dans les cas
suivants:
à l’arrêt du traitement par corticothérapie prolongée
après chirurgie pour hypercorticisme (tumeur surrénalienne ou hypophysaire)
après une antibiothérapie spécifique de la tuberculose
à l'arrêt du traitement par les anticortisoliques de synthèse
Tous les cas sont exacts
Concernant la récupé

15) Lors de la décompensation de l’insuffisance surrénale, quel est le signe qui traduit le transfert de
liquide vasculaire dans le compartiment intracellulaire:
l’installation du collapsus cardio-vasculaire souvent brutale et seulement précédée par d'une
discrète hypotension artérielle
les signes de déshydratation: faciès creusé, hypotonie des globes oculaires, pli cutané,
température 40
les troubles digestifs avec douleurs épigastriques, intolérance digestive, nausées,
vomissements et diarrhée
les troubles neurologiques variable (délire ou adynamie, asthénie intense, douleurs
musculaires) évoluant vers le coma sans signes de localisation
Lors de la décompens

16) Lequel des médicaments suivants est utilisé dans le traitement de l’insuffisance surrénale aigue?
Tétracosactide
Dihydrotestotérone
Vasopressine
Hydrocortisone
Tous
Lequel des médicamen

17) Un patient prend la prednisone 40 mg par jour pendant 6 mois. Il l’arrête brusquement, lequel
des affections suivantes peut apparaitre:
Myopathie

3
Diabète
Infection
Psychose
Insuffisance surrénale
Un patient prend la

18) Lequel des médicaments suivants peut être utilisé en vue de diagnostic d’insuffisance surrénale?
Desmopressine
Climaston
Captopril
Tétracosactide
Tous
Lequel des médicamen

19) Concernant la prise en charge de l'insuffisance surrénale aigue: Hospitalisation car urgence
vitale, quel est le point dans la démarche suivante n’est pas justifié:
Le traitement sera débuté en urgence sans attendre les résultats des examens
complémentaires
Mise en condition : pose d'une voie veineuse pour réhydratation par du sérum physiologique
avec environ 3 à 5L sur 24 h
Traitement substitutif: hydrocortisone IV en bolus de 50 mg puis 200 mg/24 h
Traitement du facteur déclenchant
Aucun
Concernant la prise

20) Concernant le traitement curatif de l’ostéporose, en association avec la supplémentation


vitamino-calcique, lequel de ces médicaments que l’on juge présenter un risque grave comme effets
secondaires si un bilan préthérapeutique est manqué:
le tériparatide
le ranélate de strontium
le traitement hormonal substitutif
lesbiphosphonates
la calcitonine
Concernant le traite

21) Dans la stratégie thérapeutique de l’ostéoporose, quel est le médicament qui répond au principe
préconisant d'agir sur les déterminants majeurs de l'ostéoporose: stimuler de la formation osseus
l’œstrogénothérapie substitutive
la supplémentation vitamine .D calcium
les bisphosphonates
la parathormone
le ranélate de strontium
Dans la stratégie th

22) Dans la stratégie thérapeutique de l’ostéoporose, quel est le médicament qui répond au principe
préconisant d'agir sur les déterminants majeurs de l'ostéoporose: stabiliser la densitométrie osseus
l’œstrogénothérapie substitutive
la supplémentation vitamine .D calcium
les bisphosphonates
la parathormone
le ranélate de strontium

4
Dans la stratégie th

23) Dans la stratégie thérapeutique de l’ostéoporose, quel est le médicament qui répond au principe
préconisant d'agir sur les déterminants majeurs de l'ostéoporose: bloquer le processus de destruction
osseus
l’œstrogénothérapie substitutive
la supplémentation vitamine .D calcium
les bisphosphonates
la parathormone
le ranélate de strontium
Dans la stratégie th

24) Une femme de 45 ans en préménopause est concernée du fait de son histoire familiale
d’ostéoporos Sa densitométire récente est T score 1.0. Choisir un traitement approprié parmi les
suivants:
estrogénothérapie substitutive
supplémentation en vitamine .D et calcium
hormonothérapie substitutive (par estroprogestatifs)
teriparatide en injection
calcitonine en spray
Une femme de 45 ans

25) Un pharmmacien reçoit une prescription de Fosamax 75 mg pour la prévention de l’ostéoporose


avec les instructions à la patiente concernant le mode de la pris Les choix ci-dessous ne sont pas
entierement corrects, sauf : I- La dose de Fosamax est de 1 comp par semaine II- Fosamax doit être
pris au moins 30 mn avant le repas III- La patient peut se coucher une heure après chaque
administration de Fosamax
I
I + II
I + II + II
I + III
II + III
Un pharmmacien reçoi

26) Quelle est la dose recommandée de calcium orale par jour pour l’adulte :
250-500 mg
300-600 mg
500-1,000 mg
1,000-1,500 mg
1,500- 2,000 mg
Quelle est la dose r

27) Lequel des médicaments suivants est considéré comme le choix de 1ère intention pour le
traitement de l’ostéoporose postménopausique:
Alendronate
Calcitonine
Minidril
Tériparatide
Tous
Lequel des médicamen

28) Lequel des médicaments suivants est présenté en forme spray nasale et injectable:

5
Raloxifène
Alendronate
Teriparatide
Calcitonine
Minidril
Lequel des médicamen

29) Lequel des médicaments suivants n’augmente pas le risque d’ostéoprose :


Depo-Provera
Tamoxifène
Glucocorticoides
Lévothyroxine
Tous
Lequel des médicamen

30) Quelle est la dose recommandée de Raloxifène dans la prévention et le traitement de


l’ostéoporose post-ménopausique:
10 mg
15 mg
40 mg
60 mg
120 mg
Quelle est la dose r

31) L’accès goutteux est due à la présence des cristaux d’urate dans l’articulation entrainant la
phagocytose et l’inflammation. La lutte contre la douleur doit être précoce, et on aura le choix un de
ces médicaments, sauf:
la colchicine
la phénylbutazone
l'indométacine
les glucocorticoides
L’accès goutteux est

32) L’accès goutteux est due à la présence des cristaux d’urate dans l’articulation entrainant la
phagocytose et l’inflammation et donc la douleur. Lequel de ces médicaments n’est pas un anti-
inflammatoire:
la colchicine
la phénylbutazone
l'indométacine
les glucocorticoides
L’accès goutteux est

33) L’accès goutteux est due à la présence des cristaux d’urate dans l’articulation entrainant la
phagocytose et l’inflammation et donc la douleur. Lequel de ces médicaments est très efficace mais
pourvu d’accident hématologique élevé:
la colchicine
la phénylbutazone
l'indométacine
les glucocorticoides
L’accès goutteux est

6
34) Dans la stratégie thérapeutique, les propositions suivantes concernent le traitement de l’accès de
goutte, sauf:
Le traitement de l’accès de goutte doit être réalisé le plus précocement possible
Le médicament choisi doit être prescrit à fortes doses dès le 1er jour
Les jours suivants, les doses seront diminuées en fonction des résultats
Il est important d'adapter les doses en fonction de la persistance de l'inflammation goutteuse
Le traitement hypo-uricémiant doit entamer en même temps que le traitement de l’accès
Dans la stratégie th

35) Ces médicaments peuvent être utilisés comme hypo-uricémiants à toutes les doses et les doses
doivent être progressives, sauf:
l’allupurinol
le feboxostat
l’aspirine
le probénicide
la benzbromarone
Ces médicaments peuv

36) Concernant le traitement de la goutte, laquelle de ces propositions qui n’entre pas dans la liste
des conseils au patient:
Nécessité d’un traitement hypo-uricémiant à vie
Régime alimentaire: aliments prurinogènes limités et lipides réduits
Boissons alcoolisées supprimées
Suppression du tabac
Exercice musculaire modéré
Concernant le traite

37) Lequel des symptomes suivants de la goutte n’est complètement pas justifié:
l’accès se fait pendant la nuit
la présence de l’hyperuricémie
la forme typique est au niveau de l’articulation métatarso-phalangienne
la présence des cristaux biréfringents dans le liquide synovial de l’articulation atteinte
la présence des tophus dans la phase chronique au niveau de l’articulation atteinte Tous
Lequel des symptomes

38) Tous les effets suivants sont en rapport avec les actions de l’allupurinol (Zyloric*), sauf :
il diminue la formation de l’acide urique par l’inhibition de la xanthine oxydase
il ne nécessite pas l’ajustement de dose chez les patients avec insuffisance rénale
les réactions cutanées tel que le Syndrome de Steven-Johnson a été rapporté lors de son
utilisation
il ne doit pas être utilisé pour le traitement de la crise de goutte
Tous les effets suiv

39) Le(s)quel (s) des médicaments suivants présente(nt) un danger potentiel d’interactions avec
Zylorc?
I- Amoxicilline
II- Imuran
III- Essidrex
I seulement
I et II seulement
I, II et III seulement

7
III seulement
Le(s)quel (s) des mé

8
1) Comment pouvez-vous mettre en evidence une paralysie faciale chez un patient comateux?
-Signe de Charles Bell
-Signe des cils de source
-Manoevre de Pirre-Marie et Foix
-Fumer la pipe
-Asymétrie faciale
Comment pouvez-vous

2) Quel est la portion du trajet de nerf facial qui est attaint pour l’inflammation de glande parotide?
Portion cérébrale
Portion intracérébrale
Portion intrapétreux
Portion extrapétreux
Portion angle pontocérébelleux
Quel est la portion

3) Quel est la portion du trajet de nerf facial qui est attaint pour la fracture de rocher?
Portion cérébrale
Portion intracérébrale
Portion intrapétreux
Portion extrapétreux
Portion angle pontocérébelleux
Quel est la portion

4) Comment pouvez-vous mettre en evidence une paralysie faciale chez un patient comateux?
Signe de Charles Bell
Signe des cils de source
Manoevre de PirrMarie et Foix
Fumer la pipe
Asymétrie faciale
Comment pouvez-vous

5) Quel examen complémentaire demandez-vous en urgence pour faire le bilan de paralysie faciale
périphérique à frigore?
IRM cérébrale
Scanner cérébrale
Consultation ORL
Scanner de rocher
Radiographe de rocher
Quel examen compléme

6) Quel est la complication otologique pour la paralysie faciale périphérique?


Spasme d’un hémiface gauche
Signe de Larme en crocodile
Choléstéatome
Hyperacousie
Hypoacousie
Quel est la complica

7) Quel étiologie ne peut être associé à une paralysie faciale périphérique?

1
Tumeur cérébrale
Tumeur de la parotïde
Traumatisme crânien
Otite moyenne aiguë
Hématome sous durale
Quel étiologie ne pe

8) Quel est l’étiologie le plus fréquence de paralysie faciale périphérique?


Paralysie à frigore
Traumatique
Infectieuse
Otologique
Maladie systémique
Quel est l’étiologie

9) Quel est le signe d’inspection retrouvé habituellement devant une paralysie faciale périphérique?
Effacement des rides du front
Sourcil abaissé
Raréfraction du clignement
Effacement du ligne nasogénien
Lagophtalmie
Quel est le signe d’

10) Comment on peut différencier la paralysie faciale périphérique et central? Paralysie faciale
périphérique:
Touche le territoire inférieur
Touche le territoire inférieur que le supérieur
Dissociation automatico-volontaire
Pas de Dissociation automatico-volontaire
Déficit moteur homogène
Comment on peut diff

11) Quand on ne fait pas de TDM/IRM cérébrale devant paralysie faciale périphérique?
Paralysie faciale périphérique isolée
Présence de signe neurologique de localisation
Forme progressive ou récidivant
Formes graves non régressive
Forme syndromique
Quand on ne fait pas

2
1) Mademoiselle TEANG Bopha , âge de 25 ans , vient consulter pour le suivi de sa maladie
asthmatique.Son asthme est connu dès l’enfance à l’âge de 9 ans , découvert par une
symptomatologie de toux nocturne , des crises dyspnéiques typiques lors de week-end à la
campagne , ou à l’effort.Le bilan initial mettait en évidence un terrain atopique , une allergie aux
pollens et aux phanères de chat . Il n’y a pas d’arguments cliniques pour un reflux gastro-
oesophagien . Quel est le facteur déclenchant de son asthme bronchique ?
. Atmosphère chaud
. Facteur psychologique
. Infection respiratoire
. Pollens végétaux
. Reflux gastro-oesophagien
Mademoiselle TEANG B

2) Mademoiselle TEANG Bopha , âge de 25 ans , vient consulter pour le suivi de sa maladie
asthmatique.Son asthme est connu dès l’enfance à l’âge de 9 ans , découvert par une
symptomatologie de toux nocturne , des crises dyspnéiques typiques lors de week-end à la
campagne , ou à l’effort.Le bilan initial mettait en évidence un terrain atopique , une allergie aux
pollens et aux phanères de chat . Il n’y a pas d’arguments cliniques pour un reflux gastro-
oesophagienUn argument clinique typique de la crise d’asthme de Mlle TEANG Bopha est :
. Dyspnée diurne
. Dyspnée nocturne
. Fièvres et frissons
. Hémoptysies répétées
. Secousses de toux sèches
Mademoiselle TEANG B

3) Mademoiselle TEANG Bopha , âge de 25 ans , vient consulter pour le suivi de sa maladie
asthmatique.Son asthme est connu dès l’enfance à l’âge de 9 ans , découvert par une
symptomatologie de toux nocturne , des crises dyspnéiques typiques lors de week-end à la
campagne , ou à l’effort.Le bilan initial mettait en évidence un terrain atopique , une allergie aux
pollens et aux phanères de chat . Il n’y a pas d’arguments cliniques pour un reflux gastro-
oesophagienQuel test doit être fait pour affirmer un terrain atopique spécifique aux pollens pour cet
asthme bronchique ?
. Hémogramme-VS
. Sérologie dans le sang
. Transaminases dans le sang
. Test épicutané anti-pollens
. Test IDR
Mademoiselle TEANG B

4) Mademoiselle TEANG Bopha , âge de 25 ans , vient consulter pour le suivi de sa maladie
asthmatique.Son asthme est connu dès l’enfance à l’âge de 9 ans , découvert par une
symptomatologie de toux nocturne , des crises dyspnéiques typiques lors de week-end à la
campagne , ou à l’effort.Le bilan initial mettait en évidence un terrain atopique , une allergie aux
pollens et aux phanères de chat . Il n’y a pas d’arguments cliniques pour un reflux gastro-
oesophagienUn signe clinique marquant une urgence thérapeutique de la crise d’asthme est :
. Bradypnée
. Cyanose
. Polypnée
. Tirage
. Sueurs

1
Mademoiselle TEANG B

5) Mademoiselle TEANG Bopha , âge de 25 ans , vient consulter pour le suivi de sa maladie
asthmatique.Son asthme est connu dès l’enfance à l’âge de 9 ans , découvert par une
symptomatologie de toux nocturne , des crises dyspnéiques typiques lors de week-end à la
campagne , ou à l’effort.Le bilan initial mettait en évidence un terrain atopique , une allergie aux
pollens et aux phanères de chat . Il n’y a pas d’arguments cliniques pour un reflux gastro-
oesophagien Le traitement de la crise d’asthme simple fait habituellement appel aux :
. Antidégranulants ( Lomudal )
. Bêta 2-mimétiques en spray
. Corticoïdes inhalés ( Bécotide 250 )
. Corticoïdes retards par IM
. Corticoïdes par voie parentérale.
Mademoiselle TEANG B

6) L’asthme aigu grave se définit par :


. Dyspnée avec anémie sévère
. Dyspnée avec asphyxie
. Dyspnée avec toux
. Dyspnée avec sibilance
. Dyspnée avec hémoptysies
L’asthme aigu grave

7) Au cours d’asthme aigu grave , un seul des signes suivants n’est pas observé ?
. Diminution de murmure vésiculaire
. Expectoration abondante
. Polypnée importante
. Signes de cœur pulmonaire aigu
. Troubles de conscience
Au cours d’asthme ai

8) Devant une suspicion de crise d’asthme aux urgences , quel examen paraclinique demandez-vous
en urgence ?
. Épreuves fonctionnels respiratoires
. Gaz du sang artériel
. Mesure des Ig E toaux
. Numération de formule sanguine
. Test allergique percutané
Devant une suspicion

9) Un asthme est souvent aggravé par l’administration de :


. Atropine
. Bêta-bloquant
. Cycline
. Paracétamol
. pénicilline
Un asthme est souven

10) Quelle stratégie thérapeutique doit-on traiter immédiatement de l’asthme aigu grave ?
. Antibiotique par voie parentérale
. Bronchodilatateurs par nébulisation

2
. Corticoïdes par voie intra-veineuse
. Radiographie pulmonaire
. Tonicardiaque par voie parentérale
Quelle stratégie thé

11) Un homme de 65 ans est atteint d’une insuffisance respiratoire chronique.Depuis l’âge de 21
ans , il fume 2 paquets de cigarettes par jour. Depuis 25 ans , il tousse et crache fréquemment en
accompagnant d’une dyspnée d’effort progressivement augmentée.Actuellement il ne peut monter
qu’une dizaine de marches environ. Les épreuves fonctionnelles respiratoires effectuées montrent :
VEMS : 1 200ml (val. Théorique : 2 500ml), CV : 2 400ml ( val. Théo. :3 200ml) , CPT : 6 000ml
(val.Théo. :5 000ml) .Les gaz du sang artériels sont : PaO2 : 54 mmHg , PaCO2 : 45 mmHg , pH :
7,41 , SaO2 : 90% , bicarbonates : 32 mmol/l . Une cause de l’insuffisance respiratoire chronique est
fréquemment rencontrée au Cambodge :
. Diabète sucré de type II avec obésité
. Insuffisance cardiaque d’origine valvulaire
. Insuffisance rénale d’origine infectieuse
. Insuffisance respiratoire obstructive par tabac
. Œdème du poumon hémodynamique
Un homme de 65 ans e

12) Un homme de 65 ans est atteint d’une insuffisance respiratoire chronique.Depuis l’âge de 21
ans , il fume 2 paquets de cigarettes par jour. Depuis 25 ans , il tousse et crache fréquemment en
accompagnant d’une dyspnée d’effort progressivement augmentée.Actuellement il ne peut monter
qu’une dizaine de marches environ. Les épreuves fonctionnelles respiratoires effectuées montrent :
VEMS : 1 200ml (val. Théorique : 2 500ml), CV : 2 400ml ( val. Théo. :3 200ml) , CPT : 6 000ml
(val.Théo. :5 000ml) .Les gaz du sang artériels sont : PaO2 : 54 mmHg , PaCO2 : 45 mmHg , pH :
7,41 , SaO2 : 90% , bicarbonates : 32 mmol/l .Une insuffisance respiratoire chronique a un tableau
clinique caractéristique comme ci-dessous :
. Toux-expectorations avec cyanose
. Toux-expectorations avec fièvre prolongée
. Toux-expectoration avec dyspnée progressive
. Toux avec expectorations muco-sanglantes
. Toux sèches avec douleur thoracique
Un homme de 65 ans e

13) Un homme de 65 ans est atteint d’une insuffisance respiratoire chronique.Depuis l’âge de 21
ans , il fume 2 paquets de cigarettes par jour. Depuis 25 ans , il tousse et crache fréquemment en
accompagnant d’une dyspnée d’effort progressivement augmentée.Actuellement il ne peut monter
qu’une dizaine de marches environ. Les épreuves fonctionnelles respiratoires effectuées montrent :
VEMS : 1 200ml (val. Théorique : 2 500ml), CV : 2 400ml ( val. Théo. :3 200ml) , CPT : 6 000ml
(val.Théo. :5 000ml) .Les gaz du sang artériels sont : PaO2 : 54 mmHg , PaCO2 : 45 mmHg , pH :
7,41 , SaO2 : 90% , bicarbonates : 32 mmol/l . Parmi les propositions ci-dessous , laquelle est
associée à l’hypoxémie chronique ?
. Accident vasculaire cérébrale
. Hypertension artérielle systémique
. Œdème pulmonaire hémodynamique
. Polyglobulie post-tabagique
. Troubles des équilibres hydro-électrolytiques
Un homme de 65 ans e

14) Un homme de 65 ans est atteint d’une insuffisance respiratoire chronique.Depuis l’âge de 21
ans , il fume 2 paquets de cigarettes par jour. Depuis 25 ans , il tousse et crache fréquemment en

3
accompagnant d’une dyspnée d’effort progressivement augmentée.Actuellement il ne peut monter
qu’une dizaine de marches environ. Les épreuves fonctionnelles respiratoires effectuées montrent :
VEMS : 1 200ml (val. Théorique : 2 500ml), CV : 2 400ml ( val. Théo. :3 200ml) , CPT : 6 000ml
(val.Théo. :5 000ml) .Les gaz du sang artériels sont : PaO2 : 54 mmHg , PaCO2 : 45 mmHg , pH :
7,41 , SaO2 : 90% , bicarbonates : 32 mmol/l .Quelle est le contexte de gravité de l’insuffisance
respiratoire ?
. Dyspnée + Hypoxémie avec hypercapnie
. Dyspnée + Hypoxémie avec normocapnie
. Dyspnée + Hypoxémie avec hypocapnie
. Dyspnée + Normoxémie avec hypercapnie
. Dyspnée + Normoxémie avec hypocapnie
Un homme de 65 ans e

15) Un homme de 65 ans est atteint d’une insuffisance respiratoire chronique.Depuis l’âge de 21
ans , il fume 2 paquets de cigarettes par jour. Depuis 25 ans , il tousse et crache fréquemment en
accompagnant d’une dyspnée d’effort progressivement augmentée.Actuellement il ne peut monter
qu’une dizaine de marches environ. Les épreuves fonctionnelles respiratoires effectuées montrent :
VEMS : 1 200ml (val. Théorique : 2 500ml), CV : 2 400ml ( val. Théo. :3 200ml) , CPT : 6 000ml
(val.Théo. :5 000ml) .Les gaz du sang artériels sont : PaO2 : 54 mmHg , PaCO2 : 45 mmHg , pH :
7,41 , SaO2 : 90% , bicarbonates : 32 mmol/l . Quels est la durée quotidienne de l’oxygénothérapie à
domicile d’une insuffisance respiratoire chronique ?
. 4 à 6 heures
. 6 à 8 heures
. 8 à 10 heures
. 10 à 12 heures
. . > 12 heures
Un homme de 65 ans e

16) Un homme de 65 ans est atteint d’une insuffisance respiratoire chronique.Depuis l’âge de 21
ans , il fume 2 paquets de cigarettes par jour. Depuis 25 ans , il tousse et crache fréquemment en
accompagnant d’une dyspnée d’effort progressivement augmentée.Actuellement il ne peut monter
qu’une dizaine de marches environ. Les épreuves fonctionnelles respiratoires effectuées montrent :
VEMS : 1 200ml (val. Théorique : 2 500ml), CV : 2 400ml ( val. Théo. :3 200ml) , CPT : 6 000ml
(val.Théo. :5 000ml) .Les gaz du sang artériels sont : PaO2 : 54 mmHg , PaCO2 : 45 mmHg , pH :
7,41 , SaO2 : 90% , bicarbonates : 32 mmol/l .
L’insuffisance respiratoire aiguë est définie par :
. Une polypnée respiratoire brutale
. Une hypotension artérielle
. Une caverne sur la radiographie pulmonaire
. Une hypoxémie à PaO2 < 60 mmHg
. Une hyperleucocytose sur l’hémogramme
Un homme de 65 ans e

17) Un homme de 65 ans est atteint d’une insuffisance respiratoire chronique.Depuis l’âge de 21
ans , il fume 2 paquets de cigarettes par jour. Depuis 25 ans , il tousse et crache fréquemment en
accompagnant d’une dyspnée d’effort progressivement augmentée.Actuellement il ne peut monter
qu’une dizaine de marches environ. Les épreuves fonctionnelles respiratoires effectuées montrent :
VEMS : 1 200ml (val. Théorique : 2 500ml), CV : 2 400ml ( val. Théo. :3 200ml) , CPT : 6 000ml
(val.Théo. :5 000ml) .Les gaz du sang artériels sont : PaO2 : 54 mmHg , PaCO2 : 45 mmHg , pH :
7,41 , SaO2 : 90% , bicarbonates : 32 mmol/l .
Quel est le signe clinique montrant une insuffisance respiratoire aiguë ?
. Bradypnée
. Fièvres

4
. Frissons
. Polypnée
. Cyanose
Un homme de 65 ans e

18) Un homme de 65 ans est atteint d’une insuffisance respiratoire chronique.Depuis l’âge de 21
ans , il fume 2 paquets de cigarettes par jour. Depuis 25 ans , il tousse et crache fréquemment en
accompagnant d’une dyspnée d’effort progressivement augmentée.Actuellement il ne peut monter
qu’une dizaine de marches environ. Les épreuves fonctionnelles respiratoires effectuées montrent :
VEMS : 1 200ml (val. Théorique : 2 500ml), CV : 2 400ml ( val. Théo. :3 200ml) , CPT : 6 000ml
(val.Théo. :5 000ml) .Les gaz du sang artériels sont : PaO2 : 54 mmHg , PaCO2 : 45 mmHg , pH :
7,41 , SaO2 : 90% , bicarbonates : 32 mmol/l .
Quel est l’examen complémentaire à demander pour affirmer une insuffisance respiratoire aiguë ?
. Cholestérolémie
. Gaz du sang artériel
. Hémoculture
. Numération de formule sanguine
. Vitesse de sédimentation sanguine
Un homme de 65 ans e

19) Un homme de 65 ans est atteint d’une insuffisance respiratoire chronique.Depuis l’âge de 21
ans , il fume 2 paquets de cigarettes par jour. Depuis 25 ans , il tousse et crache fréquemment en
accompagnant d’une dyspnée d’effort progressivement augmentée.Actuellement il ne peut monter
qu’une dizaine de marches environ. Les épreuves fonctionnelles respiratoires effectuées montrent :
VEMS : 1 200ml (val. Théorique : 2 500ml), CV : 2 400ml ( val. Théo. :3 200ml) , CPT : 6 000ml
(val.Théo. :5 000ml) .Les gaz du sang artériels sont : PaO2 : 54 mmHg , PaCO2 : 45 mmHg , pH :
7,41 , SaO2 : 90% , bicarbonates : 32 mmol/l .
Quel est le contexte clinique de gravité de l’insuffisance respiratoire ?
. Cyanose + Polypnée
. Cyanose + Hyperleucocytose
. Cyanose + PaO2 < 65 mmHg
. Cyanose + pH < 7,30
. Cyanose + Hypoglycémie
Un homme de 65 ans e

20) Un homme de 65 ans est atteint d’une insuffisance respiratoire chronique.Depuis l’âge de 21
ans , il fume 2 paquets de cigarettes par jour. Depuis 25 ans , il tousse et crache fréquemment en
accompagnant d’une dyspnée d’effort progressivement augmentée.Actuellement il ne peut monter
qu’une dizaine de marches environ. Les épreuves fonctionnelles respiratoires effectuées montrent :
VEMS : 1 200ml (val. Théorique : 2 500ml), CV : 2 400ml ( val. Théo. :3 200ml) , CPT : 6 000ml
(val.Théo. :5 000ml) .Les gaz du sang artériels sont : PaO2 : 54 mmHg , PaCO2 : 45 mmHg , pH :
7,41 , SaO2 : 90% , bicarbonates : 32 mmol/l .
Quel mode d’inhalation vous semble le mieux adapté dans l’oxygénothérapie d’une insuffisance
respiratoire aiguë ?
. Cathéter transtrachéal
. Lunette nasale
. Masque naso-baccale
. Sonde nasopharyngée
. Trachéotomie
Un homme de 65 ans e

21) Mademoiselle TEANG Bopha , âge de 25 ans , vient consulter pour le suivi de sa maladie

5
asthmatiqu Son asthme est connu dès l’enfance à l’âge de 9 ans , découvert par une symptomatologie
de toux nocturne , des crises dyspnéiques typiques lors de week-end à la campagne , ou à l’effort. Le
bilan initial mettait en évidence un terrain atopique , une allergie aux pollens et aux phanères de
chat . Il n’y a pas d’arguments cliniques pour un reflux gastro-oesophagien .
Quel est le facteur déclenchant de son asthme bronchique ?
Atmosphère chaud
Facteur psychologique
Infection respiratoire
Pollens végétaux
Reflux gastro-oesophagien
Mademoiselle TEANG B

22) Mademoiselle TEANG Bopha , âge de 25 ans , vient consulter pour le suivi de sa maladie
asthmatiqu Son asthme est connu dès l’enfance à l’âge de 9 ans , découvert par une symptomatologie
de toux nocturne , des crises dyspnéiques typiques lors de week-end à la campagne , ou à l’effort. Le
bilan initial mettait en évidence un terrain atopique , une allergie aux pollens et aux phanères de
chat . Il n’y a pas d’arguments cliniques pour un reflux gastro-oesophagien .
Un argument clinique typique de la crise d’asthme de Mlle TEANG Bopha est :
Dyspnée diurne
Dyspnée nocturne
Fièvres et frissons
Hémoptysies répétées
Secousses de toux sèches
Mademoiselle TEANG B

23) Mademoiselle TEANG Bopha , âge de 25 ans , vient consulter pour le suivi de sa maladie
asthmatiqu Son asthme est connu dès l’enfance à l’âge de 9 ans , découvert par une symptomatologie
de toux nocturne , des crises dyspnéiques typiques lors de week-end à la campagne , ou à l’effort. Le
bilan initial mettait en évidence un terrain atopique , une allergie aux pollens et aux phanères de
chat . Il n’y a pas d’arguments cliniques pour un reflux gastro-oesophagien .
Quel test doit être fait pour affirmer un terrain atopique spécifique aux pollens pour cet asthme
bronchique ?
Hémogramme-VS
Sérologie dans le sang
Transaminases dans le sang
Test épicutané anti-pollens
Test IDR
Mademoiselle TEANG B

24) Mademoiselle TEANG Bopha , âge de 25 ans , vient consulter pour le suivi de sa maladie
asthmatiqu Son asthme est connu dès l’enfance à l’âge de 9 ans , découvert par une symptomatologie
de toux nocturne , des crises dyspnéiques typiques lors de week-end à la campagne , ou à l’effort. Le
bilan initial mettait en évidence un terrain atopique , une allergie aux pollens et aux phanères de
chat . Il n’y a pas d’arguments cliniques pour un reflux gastro-oesophagien .
Un signe clinique marquant une urgence thérapeutique de la crise d’asthme est :
Bradypnée
Cyanose
Polypnée
Tirage
Sueurs
Mademoiselle TEANG B

6
25) Mademoiselle TEANG Bopha , âge de 25 ans , vient consulter pour le suivi de sa maladie
asthmatiqu Son asthme est connu dès l’enfance à l’âge de 9 ans , découvert par une symptomatologie
de toux nocturne , des crises dyspnéiques typiques lors de week-end à la campagne , ou à l’effort. Le
bilan initial mettait en évidence un terrain atopique , une allergie aux pollens et aux phanères de
chat . Il n’y a pas d’arguments cliniques pour un reflux gastro-oesophagien .
Le traitement de la crise d’asthme simple fait habituellement appel aux :
Antidégranulants ( Lomudal )
Bêta 2-mimétiques en spray
Corticoïdes inhalés ( Bécotide 250 )
Corticoïdes retards par IM
Corticoïdes par voie parentérale
Mademoiselle TEANG B

26) sur l’asthme L’asthme aigu grave se définit par :


Dyspnée avec anémie sévère
Dyspnée avec asphyxie
Dyspnée avec toux
Dyspnée avec sibilance
Dyspnée avec hémoptysies
sur l’asthme L’asth

27) sur l’asthme Au cours d’asthme aigu grave , un seul des signes suivants n’est pas observé ?
Diminution de murmure vésiculaire
Expectoration abondante
Polypnée importante
Signes de cœur pulmonaire aigu
Troubles de conscience
sur l’asthme Au co

28) sur l’asthme Devant une suspicion de crise d’asthme aux urgences , quel examen paraclinique
demandez-vous en urgence ?
Épreuves fonctionnels respiratoires
Gaz du sang artériel
Mesure des Ig E toaux
Numération de formule sanguine
Test allergique percutané
sur l’asthme Devant

29) sur l’asthme Un asthme est souvent aggravé par l’administration de :


Atropine
Bêta-bloquant
Cycline
Paracétamol
pénicilline
sur l’asthme Un ast

30) sur l’asthme Quelle stratégie thérapeutique doit-on traiter immédiatement de l’asthme aigu
grave ?
Antibiotique par voie parentérale
Bronchodilatateurs par nébulisation
Corticoïdes par voie intra-veineuse

7
Radiographie pulmonaire
Tonicardiaque par voie parentérale
sur l’asthme Quel

31) Un homme de 65 ans est atteint d’une insuffisance respiratoire chroniquDepuis l’âge de 21 ans ,
il fume 2 paquets de cigarettes par jour. Depuis 25 ans , il tousse et crache fréquemment en
accompagnant d’une dyspnée d’effort progressivement augmentéActuellement il ne peut monter
qu’une dizaine de marches environ. Les épreuves fonctionnelles respiratoires effectuées montrent :
VEMS : 1 200ml (val. Théorique : 2 500ml), CV : 2 400ml ( val. Théo. :3 200ml) , CPT : 6 000ml
(val.Théo. :5 000ml) .Les gaz du sang artériels sont : PaO2 : 54 mmHg , PaCO2 : 45 mmHg , pH :
7,41 , SaO2 : 90% , bicarbonates : 32 mmol/l .
Une cause de l’insuffisance respiratoire chronique est fréquemment rencontrée au Cambodge :
Diabète sucré de type II avec obésité
Insuffisance cardiaque d’origine valvulaire
Insuffisance rénale d’origine infectieuse
Insuffisance respiratoire obstructive par tabac
Œdème du poumon hémodynamique
Un homme de 65 ans e

32) Un homme de 65 ans est atteint d’une insuffisance respiratoire chroniquDepuis l’âge de 21 ans ,
il fume 2 paquets de cigarettes par jour. Depuis 25 ans , il tousse et crache fréquemment en
accompagnant d’une dyspnée d’effort progressivement augmentéActuellement il ne peut monter
qu’une dizaine de marches environ. Les épreuves fonctionnelles respiratoires effectuées montrent :
VEMS : 1 200ml (val. Théorique : 2 500ml), CV : 2 400ml ( val. Théo. :3 200ml) , CPT : 6 000ml
(val.Théo. :5 000ml) .Les gaz du sang artériels sont : PaO2 : 54 mmHg , PaCO2 : 45 mmHg , pH :
7,41 , SaO2 : 90% , bicarbonates : 32 mmol/l .
Une insuffisance respiratoire chronique a un tableau clinique caractéristique comme ci-dessous :
Toux-expectorations avec cyanose
Toux-expectorations avec fièvre prolongée
Toux-expectoration avec dyspnée progressive
Toux avec expectorations muco-sanglantes
Toux sèches avec douleur thoracique
Un homme de 65 ans e

33) Un homme de 65 ans est atteint d’une insuffisance respiratoire chroniquDepuis l’âge de 21 ans ,
il fume 2 paquets de cigarettes par jour. Depuis 25 ans , il tousse et crache fréquemment en
accompagnant d’une dyspnée d’effort progressivement augmentéActuellement il ne peut monter
qu’une dizaine de marches environ. Les épreuves fonctionnelles respiratoires effectuées montrent :
VEMS : 1 200ml (val. Théorique : 2 500ml), CV : 2 400ml ( val. Théo. :3 200ml) , CPT : 6 000ml
(val.Théo. :5 000ml) .Les gaz du sang artériels sont : PaO2 : 54 mmHg , PaCO2 : 45 mmHg , pH :
7,41 , SaO2 : 90% , bicarbonates : 32 mmol/l .
Parmi les propositions ci-dessous , laquelle est associée à l’hypoxémie chronique ?
Accident vasculaire cérébrale
Hypertension artérielle systémique
Œdème pulmonaire hémodynamique
Polyglobulie post-tabagique
Troubles des équilibres hydro-électrolytiques
Un homme de 65 ans e

34) Un homme de 65 ans est atteint d’une insuffisance respiratoire chroniquDepuis l’âge de 21 ans ,
il fume 2 paquets de cigarettes par jour. Depuis 25 ans , il tousse et crache fréquemment en
accompagnant d’une dyspnée d’effort progressivement augmentéActuellement il ne peut monter
qu’une dizaine de marches environ. Les épreuves fonctionnelles respiratoires effectuées montrent :

8
VEMS : 1 200ml (val. Théorique : 2 500ml), CV : 2 400ml ( val. Théo. :3 200ml) , CPT : 6 000ml
(val.Théo. :5 000ml) .Les gaz du sang artériels sont : PaO2 : 54 mmHg , PaCO2 : 45 mmHg , pH :
7,41 , SaO2 : 90% , bicarbonates : 32 mmol/l .
Quelle est le contexte de gravité de l’insuffisance respiratoire ?
Dyspnée + Hypoxémie avec hypercapnie
Dyspnée + Hypoxémie avec normocapnie
Dyspnée + Hypoxémie avec hypocapnie
Dyspnée + Normoxémie avec hypercapnie
Dyspnée + Normoxémie avec hypocapnie
Un homme de 65 ans e

35) Un homme de 65 ans est atteint d’une insuffisance respiratoire chroniquDepuis l’âge de 21 ans ,
il fume 2 paquets de cigarettes par jour. Depuis 25 ans , il tousse et crache fréquemment en
accompagnant d’une dyspnée d’effort progressivement augmentéActuellement il ne peut monter
qu’une dizaine de marches environ. Les épreuves fonctionnelles respiratoires effectuées montrent :
VEMS : 1 200ml (val. Théorique : 2 500ml), CV : 2 400ml ( val. Théo. :3 200ml) , CPT : 6 000ml
(val.Théo. :5 000ml) .Les gaz du sang artériels sont : PaO2 : 54 mmHg , PaCO2 : 45 mmHg , pH :
7,41 , SaO2 : 90% , bicarbonates : 32 mmol/l .
Quels est la durée quotidienne de l’oxygénothérapie à domicile d’une insuffisance respiratoire
chronique ?
4 à 6 heures
6 à 8 heures
8 à 10 heures
10 à 12 heures
> 12 heures
Un homme de 65 ans e

36) sur l’insuffisance respiratoire L’insuffisance respiratoire aiguë est définie par :
Une polypnée respiratoire brutale
Une hypotension artérielle
Une caverne sur la radiographie pulmonaire
Une hypoxémie à PaO2 < 60 mmHg
Une hyperleucocytose sur l’hémogramme
sur l’insuffisance r

37) sur l’insuffisance respiratoire Quel est le signe clinique montrant une insuffisance respiratoire
aiguë ?
Bradypnée
Fièvres
Frissons
Polypnée
Cyanose
sur l’insuffisance r

38) sur l’insuffisance respiratoire Quel est l’examen complémentaire à demander pour affirmer une
insuffisance respiratoire aiguë ?
Cholestérolémie
Gaz du sang artériel
Hémoculture
Numération de formule sanguine
Vitesse de sédimentation sanguine
sur l’insuffisance r

9
39) sur l’insuffisance respiratoire Quel est le contexte clinique de gravité de l’insuffisance
respiratoire ?
Cyanose + Polypnée
Cyanose + Hyperleucocytose
Cyanose + PaO2 < 65 mmHg
Cyanose + pH < 7,30
Cyanose + Hypoglycémie
sur l’insuffisance r

40) sur l’insuffisance respiratoire Quel mode d’inhalation vous semble le mieux adapté dans
l’oxygénothérapie d’une insuffisance respiratoire aiguë ?
Cathéter transtrachéal
Lunette nasale
Masque naso-baccale
Sonde nasopharyngée
Trachéotomie
sur l’insuffisance r

41) Mme KOLAP , âgée de 45 ans , domiciliée à Saang de la province de Kandal , est adressée aux
urgence pour troubles de conscience récente avec une fièvre majeure . L’examen initial montre :
température à 40oC , TA :80/35 mmHg , FC : 135/mn ,FR : 28/mn SpO2 : 91% , glycémie
capillaire : 16 mmol/l (N : 3,9 -5,6) . La patiente est ininterrogeable , ses propos sont confus .A la
stimulation douloureuse , la patiente réagit de manière adaptée . Il n’existe pas de signe de
localisation , ni de raideur méningée . On note une cyanose des extrémités et des marbrures des
genoux .L’auscultation cardiaque et pulmonaire est normale .L’abdomen est un peu distendu ,
douloureux . Il n’y a pas d’éruption , ni de lésion cutanée . La bandelette urinaire montre : glycosurie
( + ), présence de leucocytes et de nitrites Le diagnostic de sepsis sévère repose sur des syndromes et
des signes suivants mais cochez-vous ceux qui n’existent pas dans ce cas ?
Cyanose des extrémités , marbrures des genoux
Fièvre majeure à 40oC , frissons
Hypotension artétrielle, tachycardie
Râles crépitants , sous crépitants et arythmie à l’auscultation
Somnolence , polypnée à FR : 28/mn
Mme KOLAP , âgée de

42) Mme KOLAP , âgée de 45 ans , domiciliée à Saang de la province de Kandal , est adressée aux
urgence pour troubles de conscience récente avec une fièvre majeure . L’examen initial montre :
température à 40oC , TA :80/35 mmHg , FC : 135/mn ,FR : 28/mn SpO2 : 91% , glycémie
capillaire : 16 mmol/l (N : 3,9 -5,6) . La patiente est ininterrogeable , ses propos sont confus .A la
stimulation douloureuse , la patiente réagit de manière adaptée . Il n’existe pas de signe de
localisation , ni de raideur méningée . On note une cyanose des extrémités et des marbrures des
genoux .L’auscultation cardiaque et pulmonaire est normale .L’abdomen est un peu distendu ,
douloureux . Il n’y a pas d’éruption , ni de lésion cutanée . La bandelette urinaire montre : glycosurie
( + ), présence de leucocytes et de nitrites Le critère du traitement préventif individuel pour éviter la
septicémie est :
Antibiothérapie prolongée
Hospitalisation au service spécialisé
Mesures strictes d’asepsie
Répétition des hémocultures de contrôle
Répétition de radio-pulmonaire de contrôle
Mme KOLAP , âgée de

10
43) sur la septicémie/le choc septique Parmi les propositions suivantes , lesquelles sont exactes ?
En cas de sepsis sévère , une antibiothérapie doit être mise en place après l’identification du
micro-organisme sur les prélèvements réalisés. A
Le choc septique est défini , au cours d’un sepsis sévère , par une hypotension artérielle ne
répondant pas au remplissage vasculaire . B
L’existance d’une oligo-anurie , des troubles de conscience ou d’une polypnée sont , chez un
patient en état de choc , spécifiques d’origine infectieuse
Un choc septique avec douleur lombaire évoque une pyélonéphrite avec tension artérielle
normale . D
Un choc septique associé à un purpura extensif et nécrotique évoque en premier lieu une
endocardite infectieuse . E
sur la septicémie/le

44) sur la septicémie/le choc septique Quel est le signe clinique typique du choc septique ?
Cyanose des extrémités
Dyspnée respiratoire
Fièvre majeure
Hypotension artérielle
Tachycardie sinusale
sur la septicémie/le

45) sur la septicémie/le choc septique Quel est l’examen complémentaire indispensable et urgent
dans le choc septique ?
Hémogramme
Hémoculture
Ionogramme
Rx thorax
Scanner E
sur la septicémie/le

46) sur la septicémie/le choc septique Quel est le signe clinique que l’on doit utiliser tout de suite des
antibiotiques sans attendre les résultats paracliniques devant un état septicémique ?
Anurie avec douleur abdominale
État de confusion mentale
Fièvre majeure à To > 40o
Polypnée à FR > 30/mn
Tachycardie à FC > 120/mn
sur la septicémie/le

47) sur la septicémie/le choc septique L’antibiothérapie dans le traitement d’un état septique sévère ,
indiquez la proposition fausse ?
Double ou triple
Par voie parentérale
Synergique et bactéricide
En attendant les résultats de l’antibiogramme
Donnée tout de suite sur les germes suspectés
sur la septicémie/le

48) Définir la septicémie ?


- Céphalées + Convulsion

11
- Frissons + Tachycardie
- Fièvre + Hyperleucocytose
- Hypotension + Polypnée
- Cyanose + Sueurs
Définir la septicémi

49) Qu’est-ce que c’est le choc septique ?


- Hypotension artérielle + état comateux
- Hypotension artérielle + Fièvre avec frissons
- Hypotension artérielle + Hyperleucocytose
- Sepsis sévère + Hypotension malgré remplissage vasculaire
- Hypotension artérielle + Polypnée avec cyanose .
Qu’est-ce que c’est

50) Quelle est l’affirmation exacte concernant les sepsis bactériémiques à bacille à Gram négatif ?
- Ne sont jamais d’origine hospitalière
- La porte d’entrée est souvent cutanée
- La plus redoutable de leurs complications est le choc septique
- L’ictère est un symptôme constant
- L’hyperleucocytose est toujours constant .
Quelle est l’affirma

51) Le diagnostic de certitude d’une septicémie repose sur :


- Les signes fonctionnels
- Les signes physiques
- La numération de formule sanguine
- Le bilan sérologique
- Les hémocultures .
Le diagnostic de cer

52) Quelle est la principale porte d’entrée des septicémies à bacille Gram négatif ?
- Porte d’entrée cutanée
- Porte d’entrée digestive
- Porte d’entrée urinaire
- Porte d’entrée sexuelle
- Toutes les portes d’entrée .
Quelle est la princi

53) Quelle est la principale porte d’entrée des septicémies à cryptococcose ?


- Porte d’entrée cutanée
- Porte d’entrée digestive
- Porte d’entrée urinaire
- Porte d’entrée pulmonaire
- Porte d’entrée sexuelle .
Quelle est la princi

54) L’endotoxine bactérienne :


- Est une substance thermolabile
- Est un constituant présent chez toutes les bactéries
- Provoque la formation d’anticorps protecteurs

12
- Est libérée seulement par la lyse bactérienne
- Est responsable de diarrhées toxiniques .
L’endotoxine bactéri

55) Lors d’une infection invasive bactérienne à pyogène, quelle est l’anomalie biologique sanguine
observée ?
- Anémie hémolytique
- Anémie macrocytaire
- Polynucléose neutrophile
- Éosinophilie
- Lymphocytose .
Lors d’une infection

56) La prévention individuelle , le plus mieux chez le sujet âgé , pour éviter la septicémie est :
- Hospitalisation au service spécialisé
- Antiseptiques préventifs
- Antibioprophylaxie
- Examen biologique de surveillance
- Application de la vaccination .
La prévention indivi

57) En présence d’orientation étiologique , quelle est l’antibiothérapie non recommandée en cas
d’état septique ?
- En contexte communautaire : C3G + Aminoside
- En contexte nosocomial : Bêta-lactamine large spectre + Aminoside
- Une seule antibiothérapie mais à forte dose
- En cas de suspicion d’anaérobies : + Métronidazole
- En cas de suspicion de staphylocoque doré : + Glycopeptide
En présence d’orient

58) La fréquence respiratoire normale chez sujet sain est comprise entre :
- 8 à 12 mouvements / minute
- 12 à 16 mouvements / minute
- 16 à 20 mouvements / minute
- 20 à 25 mouvements / minute
- 25 à 30 mouvements / minute
La fréquence respira

59) Le syndrome de détresse respiratoire aigu est basé sur :


- Polypnée
- Bradypnée
- Hypotension artérielle
- PaO2 < 60 mmHg
- Hyperleucocytose .
Le syndrome de détre

60) Quelle est le signe associé à l’hypoxémie chronique ?


- Hypertension artérielle
- Dyspnée
- Palpitation
- Hyperleucocytose

13
- Polyglobulie .
Quelle est le signe

61) Les signes cliniques graves de l’insuffisance aiguë est :


- Dyspnée + FC > 120 /mn
- Palpitation + FR > 30 /mn
- Cyanose + Sueurs
- Fièvre à 40oC + Frissons
- Tachycardie + tirage .
Les signes cliniques

62) 5- Au cours d’une BPCO évoluée, quel est le critère le plus fiable pour indiquer une oxygénothérapie à domicile à
faible débit de 15 h / jour ?
- Dyspnée d’effort invalidante
- Cyanose permanente des extrémités
- Un VEMS/CV inférieur à 30%
- Un VEMS inférieur à 1000ml
- Une PaO2 à l’état stable inférieur à 55 mmHg .

5- Au cours d’une BP

63) Lors d’une embolie pulmonaire de moyenne gravité , l’examen du gaz du sang montre :

- Une hypoxémie avec hypercapnie

- Une hypoxémie avec normocapnie

- Une hypoxémie avec hypocapnie

- Une normoxémie avec hypercapnie

- Aucune de ces perturbations .

Lors d’une embolie p

64) La prise en charge thérapeutique d’une détresse respiratoire aiguë est :

- Hospitalisation au service spécialisé

- Antibiothérapie à large spectre

- Tonicardiaque injectable

- Corticoïde injectable

- Oxygénothérapie à haute concentration .

La prise en charge t

65) Quel mode d’inhalation vous semble le mieux adapté dans l’oxygénothérapie d’une insuffisance respiratoire

chronique ?

-Masque bucco-nasal

- Sonde naso-pharyngée

- Lunettes nasales

14
- Cathéter transtrachéal

- Autres réponses .

Quel mode d’inhalati

66) Quel débit de l’oxygénothérapie utilisez-vous dans l’insuffisance respiratoire chronique ?

- 0,5 à 1 litre/mn

- 1 à 2 litre/mn

- 2 à 4 litre/mn

- 4 à 8 litre/mn

- 8 à 12 litre/mn

Quel débit de l’oxyg

67) Quel débit de l’oxygénothérapie utilisez-vous dans l’insuffisance respiratoire aiguë ?

- 0,5 à 1 litre/mn

- 1 à 2 litre/mn

- 2 à 4 litre/mn

- 4 à 6 litre/mn

- 6 à 12 litre/mn

Quel débit de l’oxyg

68) Quel est le signe clinique caractéristique de l’asthme ?

- La fièvre prolongée

- La dyspnée respiratoire

- Des sibilants à l’auscultation

- Des crépitants à l’auscultation

- Des hémoptysies répétées

Quel est le signe cl

69) Un seul signe physique caractéristique montre la gravité de l’asthme :

- Des râles crépitants et sous crépitants

- Des râles sibilants et ronflants

- La silence à l’auscultation

- La polypnée respiratoire

- Des toux avec expectorations

Un seul signe physiq

70) Devant un asthme aigu grave , quel l’examen paraclinique demandez-vous en urgence ?

- La radiographie pulmonaire

15
- La mesure du débit de pointe

- Les épreuves fonctionnelles respiratoires

- La mesure des IgE totaux

- La mesure de gaz du sang artériel .

Devant un asthme aig

71) La confirmation du diagnostic d’un asthme repose sur des critères suivants :

- Fonctionnels

- Cliniques

- Radiologiques

- Biologiques

- Fonctionnels et radiologiques .

La confirmation du d

72) En cas de crise d’asthme , quelle est la thérapeutique la plus rapide efficace ?

- Injection IV d’hémisuccinate d’hydrocortisone

- Injection IV de théophylline

- Une bouffée de bêta 2-mimétique en spray

- Une bouffée de chromoglycate de sodium

En cas de crise d’as

73) Quel est le médicament de base utilisé dans le traitement de fond de l’asthme ?

- Oxitropium bromure

- Bricanyl inhaler

- Ventoline spray

- Corticoïdes inhalées

- Théophylline en comprimé .

Quel est le médicame

74) Quel est le médicament de base utilisé dans le traitement de la crise d’asthme ?

- Antibiotiques

- Antiallergiques

- Corticoïdes

- Bronchodilatateurs

- Analgésiques .

Quel est le médicame

75) La posologie moyenne utilisée chez l’adulte non tabagique pour obtenir une théophyllinémie dans la zône

16
d’efficacité préventive est d’environ :

- 3 à 6 mg/kg/j.

- 6 à 10 mg/kg/j

- 10 à 15 mg/kg/j

- 15 à 25 mg/kg/j

- 25 à 30 mg/kg/j

La posologie moyenne

76) Parmi les propositions suivantes , laquelle est une contre-indication à la théophylline ?

- Enfant d’âge inférieur à 30 mois

- Insuffisance cardiaque

- Insuffisance rénale

- Myasthénie

- Antécédents comitiaux .

Parmi les propositio

77) Le traitement d’une crise d’asthme simple fait habituellement appel aux :

- Corticoïdes inhalées (Bécotide 250)

- Corticoïdes retards par IM

- Bêta2-mimétiques en spray

- Antidégranulants (Lomudal)

- Corticoïdes par voie parentérale .

Le traitement d’une

78) Mme KOLAP , âgée de 45 ans , domiciliée à Saang de la province de Kandal , est adressée aux urgence pour troubles

de conscience récente avec une fièvre majeure . L’examen initial montre : température à 40oC , TA :80/35 mmHg , FC :

135/mn ,FR : 28/mn SpO2 : 91% , glycémie capillaire : 16 mmol/l (N : 3,9 -5,6) . La patiente est ininterrogeable , ses

propos sont confus .A la stimulation douloureuse , la patiente réagit de manière adaptée . Il n’existe pas de signe de

localisation , ni de raideur méningée . On note une cyanose des extrémités et des marbrures des genoux .L’auscultation

cardiaque et pulmonaire est normale .L’abdomen est un peu distendu , douloureux . Il n’y a pas d’éruption , ni de lésion

cutanée . La bandelette urinaire montre : glycosurie ( + ), présence de leucocytes et de nitrites

Quelle est la principale porte d’entrée de cette septicémie probablement à pneumocoque ?

. Porte d’entrée cutanée

. Porte d’entrée digestive

. Porte d’entrée pulmonaire

. Porte d’entrée urinaire

. Porte d’entrée sexuelle

17
Mme KOLAP , âgée de

79) Mme KOLAP , âgée de 45 ans , domiciliée à Saang de la province de Kandal , est adressée aux urgence pour troubles

de conscience récente avec une fièvre majeure . L’examen initial montre : température à 40oC , TA :80/35 mmHg , FC :

135/mn ,FR : 28/mn SpO2 : 91% , glycémie capillaire : 16 mmol/l (N : 3,9 -5,6) . La patiente est ininterrogeable , ses

propos sont confus .A la stimulation douloureuse , la patiente réagit de manière adaptée . Il n’existe pas de signe de

localisation , ni de raideur méningée . On note une cyanose des extrémités et des marbrures des genoux .L’auscultation

cardiaque et pulmonaire est normale .L’abdomen est un peu distendu , douloureux . Il n’y a pas d’éruption , ni de lésion

cutanée . La bandelette urinaire montre : glycosurie ( + ), présence de leucocytes et de nitrites.

Le diagnostic de sepsis sévère repose sur des syndromes et des signes suivants mais cochez-vous ceux qui n’existent pas

dans ce cas ?

. Cyanose des extrémités , marbrures des genoux

. Fièvre majeure à 40oC , frissons

. Hypotension artétrielle, tachycardie

. Râles crépitants , sous crépitants et arythmie à l’auscultation

. Somnolence , polypnée à FR : 28/mn

Mme KOLAP , âgée de

80) Mme KOLAP , âgée de 45 ans , domiciliée à Saang de la province de Kandal , est adressée aux urgence pour troubles

de conscience récente avec une fièvre majeure . L’examen initial montre : température à 40oC , TA :80/35 mmHg , FC :

135/mn ,FR : 28/mn SpO2 : 91% , glycémie capillaire : 16 mmol/l (N : 3,9 -5,6) . La patiente est ininterrogeable , ses

propos sont confus .A la stimulation douloureuse , la patiente réagit de manière adaptée . Il n’existe pas de signe de

localisation , ni de raideur méningée . On note une cyanose des extrémités et des marbrures des genoux .L’auscultation

cardiaque et pulmonaire est normale .L’abdomen est un peu distendu , douloureux . Il n’y a pas d’éruption , ni de lésion

cutanée . La bandelette urinaire montre : glycosurie ( + ), présence de leucocytes et de nitrites.

Quel examen complémentaire indispensable pour affirmer les germes en cause demandez-vous devant la septicémie ?

. Coproculture et API

. ECG en cas de tachycardie

. Hémocultures , ECBU

. Hémogramme et Gaz du sang

. Scanner cérébral en cas de confusion

Mme KOLAP , âgée de

81) Mme KOLAP , âgée de 45 ans , domiciliée à Saang de la province de Kandal , est adressée aux urgence pour troubles

de conscience récente avec une fièvre majeure . L’examen initial montre : température à 40oC , TA :80/35 mmHg , FC :

135/mn ,FR : 28/mn SpO2 : 91% , glycémie capillaire : 16 mmol/l (N : 3,9 -5,6) . La patiente est ininterrogeable , ses

propos sont confus .A la stimulation douloureuse , la patiente réagit de manière adaptée . Il n’existe pas de signe de

18
localisation , ni de raideur méningée . On note une cyanose des extrémités et des marbrures des genoux .L’auscultation

cardiaque et pulmonaire est normale .L’abdomen est un peu distendu , douloureux . Il n’y a pas d’éruption , ni de lésion

cutanée . La bandelette urinaire montre : glycosurie ( + ), présence de leucocytes et de nitrites.

Quel est le signe caractéristique indiquant la gravité de la septicémie ?

. Fièvre majeure avec frissons

. Hypotension avec confusion

. Oligurie avec douleur abdominale

. Polypnée avec rythme normal

. Tachycardie avec rythme régulier

Mme KOLAP , âgée de

82) Mme KOLAP , âgée de 45 ans , domiciliée à Saang de la province de Kandal , est adressée aux urgence pour troubles

de conscience récente avec une fièvre majeure . L’examen initial montre : température à 40oC , TA :80/35 mmHg , FC :

135/mn ,FR : 28/mn SpO2 : 91% , glycémie capillaire : 16 mmol/l (N : 3,9 -5,6) . La patiente est ininterrogeable , ses

propos sont confus .A la stimulation douloureuse , la patiente réagit de manière adaptée . Il n’existe pas de signe de

localisation , ni de raideur méningée . On note une cyanose des extrémités et des marbrures des genoux .L’auscultation

cardiaque et pulmonaire est normale .L’abdomen est un peu distendu , douloureux . Il n’y a pas d’éruption , ni de lésion

cutanée . La bandelette urinaire montre : glycosurie ( + ), présence de leucocytes et de nitrites

Le critère du traitement préventif individuel pour éviter la septicémie est :

. Antibiothérapie prolongée

. Hospitalisation au service spécialisé

. Mesures strictes d’asepsie

. Répétition des hémocultures de contrôle

. Répétition de radio-pulmonaire de contrôle

Mme KOLAP , âgée de

83) Parmi les propositions suivantes , lesquelles sont exactes ?

. En cas de sepsis sévère , une antibiothérapie doit être mise en place après l’identification du micro-organisme

sur les prélèvements réalisés.

. Le choc septique est défini , au cours d’un sepsis sévère , par une hypotension artérielle ne répondant pas au

remplissage vasculaire .

. L’existance d’une oligo-anurie , des troubles de conscience ou d’une polypnée sont , chez un patient en état

de choc , spécifiques d’origine infectieuse

. Un choc septique avec douleur lombaire évoque une pyélonéphrite avec tension artérielle normale .

. Un choc septique associé à un purpura extensif et nécrotique évoque en premier lieu une endocardite

infectieuse .

Parmi les propositio

19
84) Quel est le signe clinique typique du choc septique ?

. Cyanose des extrémités

. Dyspnée respiratoire

. Fièvre majeure

. Hypotension artérielle

. Tachycardie sinusale

Quel est le signe cl

85) Quel est l’examen complémentaire indispensable et urgent dans le choc septique ?

. Hémogramme

. Hémoculture

. Ionogramme

. Rx thorax

. Scanner

Quel est l’examen co

86) Quel est le signe clinique que l’on doit utiliser tout de suite des antibiotiques sans attendre les résultats paracliniques

devant un état septicémique ?

. Anurie avec douleur abdominale

. État de confusion mentale

. Fièvre majeure à To > 40oC

. Polypnée à FR > 30/mn

. Tachycardie à FC > 120/mn

Quel est le signe cl

87) L’antibiothérapie dans le traitement d’un état septique sévère , indiquez la proposition fausse ?

. Double ou triple

. Par voie parentérale

. Synergique et bactéricide

. En attendant les résultats de l’antibiogramme

. Donnée tout de suite sur les germes suspectés

L’antibiothérapie da

20
1) Quelle est la proposition exacte concernant la définition de l’hypertension artérielle ?
. Des chiffres tensionnel anormalement élevés au cours d’une consultation suffisent à retenir
le diagnostic d’hypertension artérielle
. Les valeurs de référence sont différentes chez l’homme et chez la femme
. L’hypertension artérielle systolique isolée est définie par une pression artérielle systolique
> 160 mmHg et une pression artérielle diastolique normale
. Les valeurs de références sont été obtenues lord d’une consultation au cabinet médical avec
une confirmation par deux mesures
. Chez le sujet âgé, la fréquence de l’effet blouse blanche est importante, on recommande de
mesurer la pression artérielle à la maison de ces patients en dehors du cabinet médical.
Quelle est la propos

2) Quelle est la valeur normale de la pression artérielle clinique en mmHg ?


. < 140/90 mmHg
. < 150/90 mmHg
. < 130/85 mmHg
. < 140/80 mmHg
. < 140-150/90 mmHg
Quelle est la valeur

3) Parmi les examens ci-dessous, lequel n’entre pas dans le bilan de retentissement d’une
hypertension artérielle ?
. fond d’œil
. échographie cardiaque
. électrocardiogramme
. ionogramme urinaire
. créatininémie
Parmi les examens ci

4) Quel traitement anti - hypertenseur est à instaurer, en premier intention et l’absence de contre-
indication, lorsque l’HTA est associée à une maladie coronarienne ?
. Inhibiteur de l’enzyme de conversion (IEC)
. Diurétique
. Alpha - bloquant
. Bêta - bloquant
. Antagonistes des récepteurs de l’angiotensine II (ARA-II)
Quel traitement anti

5) Quel traitement anti - hypertenseur est à instaurer, en premier intention et l’absence de contre-
indication, chez le sujet âgé hypertension systolique
. Inhibiteur de l’enzyme de conversion (IEC)
. Diurétique
. Alpha - bloquant
. Bêta - bloquant
. Antagonistes des récepteurs de l’angiotensine II (ARA-II)
Quel traitement anti

6) Quel traitement anti - hypertenseur est à instaurer, en premier intention et l’absence de contre-
indication chez une cardiopathie post IDM ?
. Inhibiteur de l’enzyme de conversion (IEC)

1
. Diurétique
. Alpha - bloquant
. Inhibiteur calcique de longue durée d’action
. Antagonistes des récepteurs de l’angiotensine II (ARA-II)
Quel traitement anti

7) Quel traitement anti - hypertenseur est à instaurer, en premier intention et l’absence de contre-
indication, chez une néphropathie diabétique type 2 à partir du stade de microalbuminurie ?
. ARA II ou IEC
. Diurétique
. Alpha - bloquant
. Bêta - bloquant
. Inhibiteur calcique de longue durée d’action
Quel traitement anti

8) Quel traitement anti - hypertenseur est à instaurer, en premier intention et l’absence de contre-
indication, chez un hypertendu ayant des antécédents d’accident vasculaire cérébral ?
. Inhibiteur calcique de longue durée d’action
. Diurétique thiazidique
. Alpha - bloquant
. Bêta - bloquant
. Antagonistes des récepteurs de l’angiotensine II (ARA-II)
Quel traitement anti

9) Quelle est l’anomalie qui peut faire évoquer la présence d’une hypertrophie ventriculaire chez un
sujet hypertendu ?
. Une espace PR à 0,28
. Un BAV 3
. Une ischémie sous épicardique à l’ECG
. Un indice de Sokolow à 40 mm
. Un axe hyper droit
Quelle est l’anomali

10) La prescription d’un inhibiteur de l’enzyme de conversion (IEC) peut responsable :


. Hypokaliémie
. D’une toux
. D’un trouble de conduction auriculo-ventriculaire
. Des œdèmes des membres inférieurs
. D’une tachycardie
La prescription d’un

11) La prescription d’un inhibiteur calcique peut être responsable :


. d’une élévation de la créatininémie
. d’une augmentation de la fréquence cardiaque
. d’une baisse de la kaliémie
. d’une insuffisance rénale fonctionnelle
. d’une agueusie
La prescription d’un

12) Une hypertension artérielle est fréquemment retrouvée dans les pathologies suivantes, sauf :
. un diabète type II

2
. une insuffisance aortique
. une insuffisance surrénale aiguë
. une hyperthyroïdie
. des antécédents d’accident vasculaire cérébral
Une hypertension art

13) Lors de la survenue d’un accident vasculaire cérébral ischémique avec tension artérielle élevée,
on peut adopter toutes les attitudes suivantes, sauf une. Laquelle ?
. hospitalisation du sujet
. mise en place de sérum glucosé
. mise en place d’oxygène par voie nasale
. traitement antihypertenseur
. une éventuelle intubation
Lors de la survenue

14) Il est licite de faire baisser les valeurs de la pression artérielle lorsqu’elle est élevée dans les cas
suivants, sauf un. Lequel ?
. insuffisance cardiaque
. dissection aortique
. infarctus du myocarde
. accident vasculaire cérébral ischémique
. Œdème aigu des poumons
Il est licite de fai

15) Une sténose de l’artère rénale doit être suspectée devant toutes les situations suivantes sauf une.
Laquelle ?
. une hypertension artérielle résistante au traitement
. un souffle abdominal
. une hypertension artérielle récemment aggravée
. une hypertension artérielle systolique isolée
. une aggravation de la fonction rénale après introduction des IEC chez un hypertendu
Une sténose de l’art

16) Parmi les propositions suivantes, laquelle est le souvent rencontrée chez le sujet âgé
hypertendu ?
. une hypertension artérielle résistant au traitement
. une hypertension artérielle systolique
. une hypertension d’apparition récente
. une hypertension récemment aggravée
. une hypertension artérielle permanente
Parmi les propositio

17) Parmi les examens biologiques suivant, l’un d’eux ne présente aucun intérêt dans le cadre du
bilan initiale d’une hypertension artérielle permanente et modérée. Indiquez lequel.
. créatininémie
. bandelette urinaire (protéinurie, hématurie)
. bilan lipidique (cholestérol total, HDL-cholestérol, LDL-cholestérol, triglycéride)
. Albuminémie
. glycémie
Parmi les examens bi

3
18) Parmi les propositions suivantes, laquelle constitue d’un mauvais choix d’une association
thérapeutique des antihypertenseurs ?
. bêtabloquant + diurétique
. diurétique thiazidique + Inhibiteurs de l’enzyme de conversion de l’angiotensine
. bêtabloquant + inhibiteur calcique de type dihydropyridine
. inhibiteur calcique et antagoniste des récepteurs de l’angiotensine II
. Inhibiteurs de l’enzyme de conversion de l’angiotensine + antagoniste des récepteurs de
l’angiotensine II
Parmi les propositio

19) Parmi les propositions suivantes, laquelle constitue la valeur en mmHg de l’hypertension
artérielle modérée selon la classification de l’OMS.
. systolique ≥ 140, diastolique ˂90
. systolique ≥ 180, diastolique ≥ 110
. systolique = 160-179, diastolique =100-109
. systolique = 130-139, diastolique =85-89
. systolique =149-159, diastolique 90-99
Parmi les propositio

20) Parmi les propositions suivantes, laquelle est correcte dans le traitement de l’hypertension
artérielle associée à une maladie coronarienne
. bêtabloquant + inhibiteur calcique de longue durée d’action
. bêtabloquant + Inhibiteurs de l’enzyme de conversion de l’angiotensine
. diurétique thiazidique + Inhibiteurs de l’enzyme de conversion de l’angiotensine
. diurétique thiazidique + inhibiteur calcique de longue durée d’action
. antagoniste des récepteurs de l’angiotensine II + inhibiteur calcique
Parmi les propositio

21) Quelle est la complication inflammatoire retardée des angines à streptocoque bêta hémolytique
du groupe A :
. Amygdalite
. Aphtose buccale
. Rhumatisme articulaire aigu
. Syndrome néphrotique
. Gingivite
Quelle est la compli

22) Agents responsables du R.A.A. :


. Haemophilus influenzae
. Staphylococcus aureus
. Streptocoque .A
. Pneumocoque
. Pseudomonas aeruginosa
Agents responsables

23) Parmi les propositions suivantes concernant le mécanisme de l’atteinte de RAA, choisissez la
proposition juste :
. un effet direct du streptocoque .A
. l’origine auto-immune de la maladie
. complication de toutes les angines
. myocardite secondaire à une septicémie

4
. Arthrite infectieuse
Parmi les propositio

24) Parmi les propositions suivantes concernant le diagnostic du RAA en phage aiguë, choisissez la
proposition juste :
. hémoculture avec un antibiogramme éventuel
. dosage des anticorps antistreptococciques
. échocardiographie
. 1 critère majeur + 2 critères mineurs de Jones + une preuve d’une infection streptococcique
récent + syndrome inflammatoire
. prélèvement de la gorge pour faire la culture avec l’antibiogramme éventuel
Parmi les propositio

25) Quelle est la condition essentielle de diagnostic du RAA ?


. le syndrome inflammatoire
. la culture de la gorge positive
. le dosage des anticorps antistreptolysines positif
. souffle cardiaque à l’auscultation
. Arthrite
Quelle est la condit

26) Quelle est le traitement de la poussée du RAA ?


. Benzathine-Benzyl-pénicilline 1.200.000 UI si poids > 30 Kg une injection unique en IM
pendant 10 jours
. Erythromycine 30 mg/kg/j repartie en 3 prises au moment des repas pendant 10 jours si
allergie à la pénicilline
. Prednisone 2mg/kg/j sans dépasser 80 mg/j repartie en 3 prises au moment des repas
. l’hospitalisation + mesures générales + traitement antibiotique pour la suppression du
streptocoque .A du pharynx et traitement anti-inflammatoire pour lutter contre les phénomènes
inflammatoires
. le traitement à domicile est possible sauf dans le cas urgent
Quelle est le traite

27) Parmi les propositions suivantes concernant la prévention secondaire, choisissez la proposition
juste :
. traitement antibiotique prolongé pour assurer l’éradication rapide de tout foyer
streptococcique pharyngé et si nécessaire associé à une amygdalectomie.
. un suivi clinique et écho cardiographique pour surveiller l’évolution naturelle des
valvulopathies
. l’antibioprophylaxie secondaire par Benzathine Benzyl Pénicilline en intramusculaire
. l’association d’un traitement antibiotique prolongé pour l’éradication de tout foyer
streptococcique pharyngé + un suivi clinique et écho cardiographique + un antibioprophylaxie
secondaire par Benzathine Benzyl Pénicille
. Aucune de ces propositions n’est pas juste
Parmi les propositio

28) Parmi les propositions suivantes concernant la durée de la prévention des rechutes pour les
patients sans séquelle de cardite (groupe 0), choisissez la proposition juste :
. l’antibioprophylaxie secondaire jusqu’à l’âge de 40 ans
. l’antibioprophylaxie secondaire jusqu’à l’âge de 20 ans et/ou au minimum 5 ans
. l’antibioprophylaxie secondaire jusqu’à l’âge de 20 ans et/ou au minimum 5 ans puis à
discuter au cas par cas notamment en fonction des antécédents familiaux

5
. prolonger le traitement antibiotique à un mois dans le but d’éradiquer de tout foyer
streptococcique pharyngé
. Aucune de ces propositions n’est pas juste
Parmi les propositio

29) Quelle la proposition juste dans la prévention primaire du RAA ?


. le traitement antibiotique des angines/pharyngites/rhinopharyngites après l’âge de 4 ans
après la confirmation de l’infection du streptocoque .A par la culture du prélèvement de la gorge
. le traitement rapidement par l’antibiotique des angines/rhinopharyngites après l’âge de 4
ans d’au moins 08 jours
. le traitement seulement par l’antibiotique des angines/rhinopharyngites surinfectées
bactériennes après l’âge de 4 ans
. le traitement seulement par l’antibiotique des angines/rhinopharyngites après confirmation
de l’infection par le streptocoque .A après le résultat de la culture du prélèvement de gorge des
malades après l’âge de 4 ans
. Aucune de ces propositions n’est pas juste
Quelle la propositio

30) Parmi les propositions suivantes concernant la durée de la prévention des rechutes pour les
patients avec séquelle (groupe 1), choisissez la proposition juste :
. l’antibioprophylaxie secondaire jusqu’à l’âge de 40 ans
. l’antibioprophylaxie secondaire jusqu’à l’âge de 20 ans et/ou au minimum 5 ans
. l’antibioprophylaxie secondaire jusqu’à l’âge de 20 ans et/ou au minimum 5 ans puis à
discuter au cas par cas notamment en fonction des antécédents familiaux
. prolonger le traitement antibiotique à un mois dans le but d’éradiquer de tout foyer
streptococcique pharyngé
. Aucune de ces propositions n’est pas juste
Parmi les propositio

6
1) េតេ េពល ខះែដលេយង តវេធ Prélèvement microbiologique?
Dysurie sans fièvre
Pharyngite aigue
Pneumonies communotaires graves et hypoxémiantes
Diarrhea aigue
Abcès du foie non compliqué
េតេ េពល ខះែដលេយងត

2) េ លបំណងៃន associations les antibiotiques?


Elargir la spectre
Réduire la durrée de traitement
Synergie les bactericides et limitation les risques d’émergence de résistantes
Moins d’effets indésirables
Moins toxicities
េ លបំណងៃន associati

3) Quels sont les points suivient de l’echec antibiothérapie:


persistance des signes > 48h et/ou apparition de métastase septique et/ou extension localiser
Attendre de nouvel résultat bactériologique pour confirmer,
nouvelle infection après la guérison
réapparition au cours de la convalescente
Quels sont les point

1
1) On définit une vascularite comme :
- une atteinte de la paroi artérielle avec inflammation périartérielle
- Une inflammation des vaisseaux lymphatique
- Une inflammation de la paroi des vaisseaux avec leur calibre dilaté
- Une inflammation des vaisseaux sans altération de la paroi vasculaire
- Une inflammation des parois du vaisseau sans dilatation de la paroi vasculaire
On définit une vascu

2) Parmi les maladies suivant laquelle est classifiée dans le groupe de vascularite des artères de petit
calibre
- Henoch-schönlein pupura
- Polyarteritis nodosa
- Cutaneous polyarteritis
- Takayasu’s arteritis
- Giant cell arteritis
Parmi les maladies s

3) Les propositions suivant sont concernant l’étiologie de vascularite, laquelle est correct ?
- Infection gonococcique ne provoque pas une vascularite
- Endocardite bactérienne ne provoque pas une vascularite
- Infection à candida albicans ne provoque pas une vascularite
- Cancer ne provoque pas une vascularite
- Infection viral (hépatite .B ou .C) provoque une vascularite
Les propositions sui

4) La pathogénie des vascularites est complexe, quelle proposition est correcte ???
- Déposition des complexes immuns
- IgA ne joue aucun un rôle de la maladie
- IgE joue un rôle important de la maladie
- Médicament ne joue aucun rôle de la maladie
- Infection ne joue aucun rôle de la maladie
La pathogénie des va

5) Quelle proposition est correcte à propose de symptômes de vascularite :


- Il n’existe pas de douleur abdominale
- Il n’existe pas de fièvre
- Il n’existe pas de problème visuel
- Il n’existe pas de trouble neurologique
- Il provoque une toux chronique
Quelle proposition e

6) Quelle proposition est correcte à propose de manifestation cutanée de vascularite :


- des plaques erythemato-squameuses
- des macules hypopigmentées
- des vésicules en groupe
- des pustules sur les pommes des mains et les plante des pieds
- des purpuras palpables
Quelle proposition e

7) Pour confirmer diagnostic une vascularite cutanée on a besoins quelque fois une biopsie, quelle

1
proposition est correct :
- Biopsie au niveau une lésion chronique
- Biopsie au centre une lésion nécrotique
- Biopsie en peau saine
- Biopsie au niveau du bord une lésion fraiche
- Biopsie au niveau de la muqueuse
Pour confirmer diagn

8) Un garçon de 12 ans est consulté pour douleur abdominal, arthralgie et des taches purpuriques sur
les deux jambes. Du fait une suspicion de Henoch-Schönlein purpura son médecin a demandé des
examens complémentaires suivant, lequel est le plus correct
- IgE total
- IgE spécifique
- Prick test
- Patch test
- Test des fonctions rénales
Un garçon de 12 ans

9) Une dame de 70 ans est consultée pour céphalée depuis une semaine. Elle se plaint aussi de
double vision et des périodes de claudication des membres inférieurs. Elle a perdu 15 kg de poids en
2 mois. Quelle est la meilleure prise en charge ?
- Consultez avec ophtalmologue ou neurologue
- Préparer une biopsie de l’artère temporale
- Donnez antibiotique à forte dose
- Faites échographie doppler, puis faire biopsie de l’artère temporale
- Administration Prednisolone 1mg /kg poids, avant de biopsie de l’artère temporale
Une dame de 70 ans e

10) En cas de Henoch-Schönlein Purpura vous devriez donner corticostéroïde orale, mais pour quelle
raison ?
- Pour diminuer l’œdème rénal
- Pour prévenir la récurrent de la maladie
- Pour raccourcir la maladie
- Pour aider des douleurs articulaire et douleurs abdominales
- Pour traiter les lésions cutanées
En cas de Henoch-Sch

11) Which organ that is not affected in polyarteritisnodos?


- Lung
- Bones
- Brain
- Kidneys
- GIT
Which organ that is

12) Which one of the following is typically elevated in the serum of Wegener’s granulomatosis
patient?
- c-ANCA
- p-ANCA
- ESR
- Rheumatoid factor

2
- ASLO serology
Which one of the fol

13) Which one of the following antibody is elevated in Henochschonleinpurpurapatient?


- IgM
- IgA
- IgE
- IgG
- IgD
Which one of the fol

14) Which diseases can producegranulomatous inflammation in the blood vessel?


- Cutaneous lukocytoclastic
- Giant cell arteritis
- polyarteritisnodosa
- Wegener’s granulomatosis
- Henoch-Schonlein Purpura
Which diseases can p

15) Les propositions suivant sont concernant lupus érythémateux, laquelle est correct ?
- Est une maladie inflammatoire chronique auto-immune
- Est une infestation parasitaire chronique
- Est une infection bactérienne cutanée chronique
- Est une infection virale chronique
- Est une infection mycosique profonde
Les propositions sui

16) En cas de lupus érythémateux, le système immunitaire joue un rôle important, quelle proposition
est correcte ?
- Le système immunitaire est inactif dans le lupus érythémateux discoïde
- Le système immunitaire est inactif dans toutes les formes
- Le système immunitaire est sous actif dans toutes les formes
- Le système immunitaire réagit au démarrage et d’arrêt
- Le système immunitaire est hyperactif dans toutes les formes
En cas de lupus éryt

17) Le lupus subaiguë se manifeste par :


- Des plaques érythémateuses annulaires
- Des plaques érythémato-squameuses
- Des tâches hyperpigmentées
- Des papules inflammatoires confluence en plaques prurigineuse
- Des plaques annulaires, lichenifiées, excoriées
Le lupus subaiguë se

18) Quelle est le symptôme de lupus érythémateux, quelle proposition est correcte ?
- Douleur articulaire surtout au niveau des premières phalanges
- Plaques érythémato-squameuses qui sont disparues après expositions au soleil
- Des pustules palmo-plantaire
- Des plaques avec des bords nettes, élevés et peau saine au centre
- Des plaques érythèmato-squameuses sur la région décolleté
Quelle est le symptô

3
19) Une forte concentration d'anticorps antinucléaires dans le sérum d’un patient:
- permet d'affirmer le diagnostic de certitude de lupus érythémateux disséminé
- s'observe au cours de psoriasis rhumatoïde
- s'observe au cours de la périartérite noueuse
- Observer au cours de nombreuses connectivites
- s'observe au cours de Henoch-Schönlein purpura
Une forte concentrat

20) Quelle est anomalie hématologique dans les critères d’ACR du lupus érythémateux systémique :
- Leucocytose
- Positive de sérologie de l’hépatite
- Elévation de taux d’IgE total
- Fausse sérologie syphilitique
- Lymphocytose
Quelle est anomalie

21) Quel examen complémentaire répétez-vous pour surveiller l'évolution d'une patiente avec lupus
érythémateux disséminé ?
- Hémogramme et VS
- Transaminase et sérologie de l’hépatite
- Taux d’acide urique
- Taux lipidémie (Cholestérol et triglycéride)
- Taux hormonaux (LH, FSH)
Quel examen compléme

22) Un diagnostic de lupus est évoqué chez une malade. Elle est traitée dont par Hydroxychloroquine
(Plaquenil©). Quelle recommandation proposez- vous dans 3 -6 mois pour suivre le traitement ?
- Une consultation avec gynécologue
- Une consultation avec pneumologue
- Une consultation avec cardiologue
- Une consultation avec urologue
- Une consultation avec ophtalmologue
Un diagnostic de lup

23) Le traitement de fond à tous les lupiques est :


- Le méthotrexate
- Hydroxychloroquine
- Dapsone
- Mycophénolate mofétil
- Cyclophosphamide
Le traitement de fon

24) Le traitement en première intention pour une patiente lupique avec des atteintes neurologiques
repose essentiellement sur :
- Antibiothérapies à forte dose
- Des rétinoïdes systémiques
- Dapsone à forte dose
- Corticothérapies à forte dose
- Mycophénolate mofétil à forte dose
Le traitement en pre

4
25) Sclérodermie est caractérisé par
- dépôt excessif de l’adipocytes
- dépôt excessif de mélanocytes
- dépôt excessif de kértinocytes
- dépôt excessif de sébum
- dépôt excessif de tissue collagèneuses
Sclérodermie est car

26) La sclérodermie est définit comme :


- Une maladie contagieuse
- Une maladie touchée surtout les femmes jeunes
- Une maladie avec peau grasse
- Une maladie avec peau élastique
- La maladie provoque une transpiration forcément
La sclérodermie est

27) Le syndrome CREST tire son acronyme des différents signes cliniques :
- Calcification osseuse, phénomène de Raynaud, trouble de la motilité oesophagienne, Sueur
profuse, et Tremblement
- Calcification osseuse, phénomène de Raynaud, trouble de la motilité oesophagienne,
Sclérodactylie, et Tremblement
- Claudication, phénomène de Raynaud, trouble de la motilité oesophagienne,
Sclérodactylie, et Tremblement
- Calcinose, phénomène de Raynaud, trouble de la motilité oesophagienne, Sclérodactylie, et
Télangiectasie
- Claudication, phénomène de Raynaud, trouble de la motilité oesophagienne,
Sclérodactylie, et Télangiectasie
Le syndrome CREST ti

28) une femme 30 ans est consultée pour dyspnée, cyanose des doigts avec sclérodactylie. La
recherche des anticorps antinucléaires était positive.
Quel est le diagnostic le plus probable ?
- Lupus érythémateuse disséminé
- Malade mixte du tissu conjonctif
- Psoriasis rhumatoïde
- Dermatomyosite
- Sclérodermie systémique
une femme 30 ans est

29) Une jeune fille de 18 ans s’est présentée pour pâleur des extrémités quand elle expose au froid
avec douleur et cyanose. La jeune fille peut atteint par une maladie dans son avenir, laquelle?
- Polyarthrite rhumatoïde
- Lupus érythémateux discoïde
- Sclérodermie systémique
- Vascularite
- Dermatomyosite
Une jeune fille de 1

30) Maladie de Raynaud est caractérisé par propositions suivant. Laquelle est correcte ?
- La plupart des patients sont les hommes

5
- Le pronostic est sombre
- Les symptômes régressent à l’exposition des extrémités au froid
- Le test d’anticorps antinucleaires positif dans la maladie
- C’est une cause commune de phénomène de Raynaud
Maladie de Raynaud e

31) Quel examen paraclinique vous devrez faire en première ligne devant une patiente avec
sclérodermie systémique ?
- Hémogramme, VS et Culot urinaire
- Sérologie de l’hépatite, Transaminase, ECG
- Sérodiagnostic de Widal, ECG et CT-Scan
- Ionogramme, ECG et Radio du thorax
- Cholestérole total, Triglycéride et Sérodiagnostic de Widal
Quel examen paraclin

32) Le CREST Syndrome est une forme particulière de sclérodermie. La présence des anticorps
suivant peut compléter le tableau. Quelle proposition est correcte ?
- Anticorps anti-phospholipase A2
- Anticorps de l’hépatite .B
- Anticorps de l’hépatite .C
- Anticorps anti-thyroperoxydase
- Anticorps anti-centromères
Le CREST Syndrome es

33) La sclérodermie peut se développer au long cours des complications suivant, laquelle est
correcte ?
- Varice œsophagienne
- Polype intestinal
- Hernie hiatal
- Colite ulcéreuse
- Sténose de l’œsophage
La sclérodermie peut

34) Dans le traitement de sclérodermie systémique on peut donner des médicaments suivant, le quel
peut déclencher le phénomène de Raynaud ?
- Ibuprofène
- Méthotrexate
- Cyclosporine
- Propranolole
- Nifédipine
Dans le traitement d

35) Quelle préparation parmi les suivantes est une forme peu grasse (huile dans l’eau ?
- Les Poudres
- Les Pommades
- Les Gels
- Les Solutions
- Les Laits
Quelle préparation p

36) Parmi les produits suivant lequel est un agent kératolytique ?

6
- Crème de métronidazole
- Crème d’érythromycine
- Pommade de clobéthasole
- Crème de l’urée
- Pommade de vaseline
Parmi les produits s

37) Une fille de 18 ans se présent des plaques erythèmato-squameuses sur le corps, les plie du coudre
et sur les genoux. Vous faites diagnostic de psoriasis vulgaris.
Quelle préparation magistrale vous donnerez à la patiente ?
- Les Poudres
- Les Pommades
- Les Gels
- Les Solutions
- Les Pâtes
Une fille de 18 ans

38) Un garçon de 8 ans se présent des plaques multiple, excoriées et suintant sur les deux pieds. A
l’examen vous trouvez adénopathie dans les deux creux inguinaux. Vous avez diagnostiquez un
eczéma infecté.
Quelle préparation magistrale vous donnerez au patient ?
- Les Poudres
- Les Pommades
- Les Gels
- Les laits
- Les Pâtes
Un garçon de 8 ans s

39) Les produits suivants sont dermocorticoïdes. Lequel est classé comme dermocorticoïde classe 3
(classification international)?
- Clobetasol
- Betamethasone
- Hydrocortisone
- Triamcinolone
- Désonid
Les produits suivant

40) Les crèmes suivantes sont utilisées pour le traitement de l’acné. Laquelle est la crème rétinoïde ?
- Brévoxyl©
- Skinoren©
- Rozex ©
- Eryderm©
- Retin-A©
Les crèmes suivantes

41) Which of the following procedures is considered to be topical treatment?


- Chemical cautery
- Chemical Peeling
- Cryosurgery
- Curettage
- Local injection of triamcinolone

7
Which of the followi

42) Toxicity of topical drug are following, tick the wrong answer:
- Stinging
- Irritation
- Allergic contact dermatitis
- Systemic absorption
- Liver toxic
Toxicity of topical

43) The following topical medicines are anti-inflammatory, tick the wrong answer
- Dithranol
- Corticosteroid
- Vitamin.D derivative
- Tacrolimus
- Urea
The following topica

44) Topical steroid has the following mechanism of actions, tick the wrong answer
- Anti-inflammatory
- Anti-proliferative
- Anti-bacterial
- Immunosuppressive
- Vasoconstriction
Topical steroid has

45) A young man work at the construction comes to you for sever pruritus. Up on examination you
found erythematous, excoriated papules and vesicle confluence to plaques on the back, chest, both
arms and also on the feet. You made a diagnosis of allergic contact dermatitis, probably contacted to
cement. What will be you appropriate treatment?
- Hydrocortisone cream twice daily
- Antibiotic cream twice daily
- Urea cream twice daily
- Oral Prednisolone, 1mg/Kg BW
- Oral acyclovir 400mg, 5 times daily
A young man work at

46) The following medicine are immunosuppressive agents. Which one is not belong to the group?
- Methotrexate
- Micophenolate mofetil
- Cyclosporine
- Cephalosporine
- Cyclophosphamide
The following medici

47) A prolong use of topical potency steroid could cause the following side effects. Which one is one
you never have seen?
- Skin bleaching
- Skin atrophy
- Telangiectasia
- Keloid

8
- Striae distansae cutis
A prolong use of top

48) The following topical medications are antibacterial. Please tick the wrong one,
- Cyclosporine lotion
- Erythromycine gel
- Clindamycine gel
- Tetracycline gel
- Fusidic acide cream
The following topica

49) The following topical medications are antifungal agents. Please tick the wrong one,
- Ketoconazole cream
- Terbinafine cream
- Metronidazole gel
- Miconazole gel
- Fluconazole gel
The following topica

9
1) La toux sèche sont origines suivant, soif un lequel ?
Toux d’origine de crise d’asthme
Toux d’origine d’exacerbation de BPCO
Toux d’origine cardiaque
Toux d’origine nerveuse
Toux d’origine métabolique
La toux sèche sont o

2) Parmi les médicaments suivant utilisent pour la toux expectoration, soif un lequel ?
Acetylcystéine
Bromhexine
Dextrométhorphane
Ambroxol
Erdostéine
Parmi les médicament

3) Les antitussifs centraux ne sont pas utilisés dans les cas suivant :
Sujet âgé ou Détresse respiratoire sévère
Sujet âgé ou Atteint neurologique
Détresse respiratoire sévère Sujet ou Atteint neurologique
Asthme ou BPCO sévère
Patient intubé
Les antitussifs cent

4) Toutes les recherches étiologiques sont restées négatives. Bien que le patient soit ambulatoire et
qu'il ne présente pas de phlébite clinique, vous cherchez à éliminer une éventuelle embolie
pulmonaire.
Quel(s) examen(s) peut (peuvent) vous aider ?
Une scintigraphie pulmonaire
Une angiopneumographie
Une pléthysmographie des membres inférieurs
Une phlébographie des membres inférieurs
Une artériographie bronchique
Toutes les recherche

5) Les hémoptysies ont repris, devenant alarmantes, avec risque asphyxique et anémique. Dans ce
cas, il est logique de prescrire de toute urgence :
Une bronchoscopie en urgence
Des substances vasopressives
Un sonde nasogastrique
Une tentative d'obturation bronchique provisoire avec une sonde de Fogarty sous fibroscopie
D
Une perfusion de corticoïdes
Les hémoptysies ont

6) La cause fréquente d’hémoptysie minime chez patient séquelle tuberculeuse


Aspergillome
Dilatation des bronches
Pneumopathie
Abcès du poumon

1
Travail forcé
La cause fréquente d

7) Traitement médicale d’hémoptysie est le moyen simple et efficace, parmi les propositions suivant
sont correct soif une, la quelle ?
Mesures thérapeutiques médical générale
Traitement vasoconstricteur: terlipressine (Glypressine*) par voie systémique
Diminution la fréquence cardiaque et baissé la tension artérielle
Traitement endoscopique: Sérum physiologique ou Sérum physiologique + adrénaline, ( ou
glypressine) Tamponnement par un cathéter à ballonnet
Activé de facteur VII coagulation
Traitement médicale

2
1) Parmi les localisations des causes de la dyschésie ci-dessous, une est correcte:
Le système nerveux central
L’intestin grêle
Le côlon droit
Le côlon transverse
La région pelvienne
Parmi les localisati

2) Une fausse diarrhée est caractérisée par de selles :


quotidienne liquides s’accompagnant de coliques.
quotidiennes liquides s’accompagnant de glaires
avec bouchon au début suivi de selles liquides après une période d’absence de quelques
jours.
liquides s’accompagnant de glaires et de sang.
dures après une période d’absence de quelques jours.
Une fausse diarrhée

3) Dans la constipation idiopathique, parmi les facteurs extra digestifs influençant négativement la
progression des selles, un est faux:
Manque activité physique
Déficit alimentaire en fibres
Ration hydrique insuffisante
Facteurs psychologiques.
Une insuffisance cardio-respiratoire
Dans la constipation

4) Parmi les signes cliniques suivants de la dyschésie, un est faux.


Des efforts de poussée,
La vertige
Une sensation de gêne au passage selles,
Une sensation d’évacuation incomplète,
Une exonération prolongée,
Parmi les signes cli

5) Parmi les complications de la constipation négligée ci-dessous, une est fausse:


Apparition des hémorroïdes
Fissure anale
Impaction fécale
Cancer rectal
Mégarectum
Parmi les complicati

6) Parmi les actions suivantes des laxatifs stimulants, une action est correcte:
Stimuler la motricité colique
Osmotique du contenu intestinal
Lubrification des selles
Formation des ballasts par retenu de l’eau dans le grêle et le côlon
Stimuler le relâchement des sphincters anaux .
Parmi les actions su

1
7) Indiquez une proposition vraie. A l’examen physique d’une diarrhée aiguë, on peut trouver:
Une réaction péritonéale à la palpation
Une sonorité diffuse à la percussion.
Un point douloureux exquis à la palpation profonde.
Des mouvements intestinaux visibles à l’inspection
La majorité des cas le ventre est normal.
Indiquez une proposi

8) A l’examen physique chez les diarrhées aiguës, la présence des signes suivants indique la gravité,
Une proposition est fausse:
Une hypothermie.
Une perte 10% du poids .
Un choc hypovolémique.
Une diarrhée hémorragique.
La presence abondante de glaire dans les selles.
A l’examen physique

9) En pratique clinique, les caractéristiques des selles suivantes indiquent une diarrhée aiguë. Une
proposition est correcte:
Selles très molles ou liquidiennes au moins 3 fois par jour.
Selles dures morcelées plus de 3 fois par jour.
Selles de consistance molle normale plus de 3 fois par jour.
Selles avec de glaires et de sang plus de 3 fois par jour.
Selles de consistance molle normale enrobées de sang 3 fois par jour.
En pratique clinique

10) Le traitement probabiliste d’emblée avec antibiotiques est indiqué dans les cas de diarrhée aiguë
suivants. Indiquez une proposition fausse.
Des gens atteints de valvulopathie.
grand vieillard avec comorbidités majeures.
syndrome septicémique.
déshydratation majeure.
Une fébricule de 38oC.
Le traitement probab

11) Malgré un traitement correct, la diarrhée persiste plus de 3 jours, quelle est votre conduite à tenir. Une proposition est
fausse
Proposer une coproculture,
un examen de selles,
un hémogramme.
La protéine .C réactive.
Une radiographie de l’abdomen sans préparation
Malgré un traitement

12) Parmi les propositions suivantes une est fausse dans le diagnostic d’un syndrome dysentérique.
Des émissions de selles de consistance normale plusieurs fois par jour avec des épreintes dans la fosse iliaque
gauche.
Des émissions de selles de consistance normale plusieurs fois par jour avec des ténesmes, sans glaires ni sang.
Des émissions de selles de consistance normale 1 fois par jour alternées des émissions de glaire et de sang.

2
Des émissions de selles liquides plusieurs fois par jour sans ténesme ni épreintes.
Parmi les propositio

13) Dans le tableau clinique d’une diarrhée aiguë banale, une proposition est correcte :
Commence par un début soudain.
S’accompagne toujours de coliques abdominales.
Est toujours grave nécessitant une hospitalisation.
S’accompagne toujours de vomissements.
La présence d’une fièvre indique l’origine bactérienne.
Dans le tableau clin

14) Un des agents pathogènes fréquemment responsables d’un syndrome dysentérique aiguë est :
Giardia intestinalis
Clostridiume difficile
Entameba histolytica
Eschericia coli.
Entameba coli
Un des agents pathog

15) A propos des facteurs de risque de la diarrhée nosocomiale, un est faux.


L’âge avancé du patient.
La présence d’un voisin diarrhéique dans la chambre
Un longue séjour d’hospitalisation
Un environnement souillé
Survient dès le premier jour d’hospitalisation
A propos des facteur

16) Les mécanismes physio-pathologiques suivants sont responsables de la diarrhée chronique. Une affirmation est
fausse.
Une augmentation de la pression osmotique du contenu intestinal.
Une augmentation de la sécrétion intestinale.
Une inflammation
Une malabsorption
Une allergie alimentaire
Les mécanismes physi

17) 4. Les médicaments suivants peuvent provoquer une diarrhée type osmotique. Une affirmation est fausse:
Le sulfate de magnésium
Le sorbitol
Le mannitol
Les topiques antiacides.
Le lopéramide
4. Les médicaments s

18) Les maladies suivantes sont considérées comme une diarrhée chronique type exsudative. Une proposition est fausse:
Une tuberculose intestinale
Une entérite régionale (localisation intestinale de la maladie de Crohn)

3
Une colite inflammatoire.
Un cancer du colon
Un déficit de lactase
Les maladies suivant

19) Les situations pathologiques suivantes peuvent provoquer une díarrhée chronique type malabsorption. Une situation
est fausse:
Une résection chirurgicale de l’estomac
Une résection chirurgicale de l’intestin grêle
Une résection chirurgicale du côlon.
Une vagotomie
Une tuberculose intestinale
Les situations patho

20) Parmi les maladies endocriniennes suivantes, une fait partie dans l’exploration initiale de la diarrhée chronique
Le diabète sucré.
La pancréatite chronique.
L’insuffisance surrénale
l’hypothyroïdie
L’hyperthyroidie
Parmi les maladies e

4
1) The most frequent cause of UGI bleeding is:
. Esophageal varices
. Peptic ulcer disease
. Angiomata
. Mallory Weiss tear
. Gastritis
The most frequent ca

2) After initial stabilization and resuscitation of the patient, each of the following Options should be
considered in the management of UGI bleeding except:
. Determine the source of bleeding
. Stop acute bleeding
. Treat the underlying abnormality
. Prevent rebleeding
. Emergency surgery.
After initial stabil

3) Endotracheal intubation for airway protection in the management of UGI bleeding should be
considered:
. in all cirrhotic patients
. in all patients with UGI bleeding
. in patients with altered mental status and ongoing hematemesis
. in patients with stable COPD
. unless it delays urgent endoscopy
Endotracheal intubat

4) A 73 year old man presents with several episodes of hematemesis. Examination shows signs of
orthostatic hypotension and melena. What is the first priority in caring for this patient?
. Nasogastric tube placement and gastric lavage.
. Resuscitation with adequate IV access and appropriate fluid and blood product fusion.
. Intravenous infusion of H2-receptor antagonists to stop the bleeding.
. Urgent upper panendoscopy.
. Urgent surgical consultation.
A 73 year old man pr

5) An important risk factor for peptic ulcer hemorrhage includes:


. Gastric acid hypersecretion
. Corticosteroid use
. Cigarette smoking
. Non-sterioidal anti-inflammatory drug use
. Ethanol consumption
An important risk fa

6) For the patient who is now stable after a severe UGI bleed associated with NSAID ingestion, and
who is found to be H.pylori positive, what is the most effective management strategy?
. Stop NSAIDS, eradicate H.pylori, PPI therapy IV
. Stop NSAIDs, full dose H2RA or daily PPI maintenance therapy
. Stop NSAIDs
. Eradicate H.pylori, full dose H2RA or daily PPI maintenance therapy
. Full dose H2-receptor antagonist or daily PPI maintenance therapy

1
For the patient who

7) For the patient with an UGI bleed and the endoscopic finding of a clean ulcer, the most
appropriate management includes:
. Endoscopic hemostasis with multipolar or heater probe or injection treatment
. Endoscopic hemostasis with combination therapy
. Emergent surgery
. Medical therapy, early refeeding, same day discharge if stable(medically) and reliable
. Medical therapy alone in a monitored setting for three days
For the patient with

8) A sixty-six-year old man presents to the emergency department with a history of one episode of
melena. Past history includes coronary artery disease, hypertension . He is on one baby aspirin daily.
An urgent upper endoscopy is negative. What is the most appropriate next step?
. UGI series with small bowel follow-through
. Colonoscopy
. Angiography
. Red blood cell tagged technetium scan
. Abdominal CT scan with contrast
A sixty-six-year old

9) A fifty-eight year old female patient presents to the emergency department with a 24-hour history
of several hematemesis. She is found to be hypotensive and anemic. Resuscitative measures are
instituted. What is the most appropriate next step?
. Urgent gastroscopy
. Anoscopic examination
. Colonoscopic examination
. Scintigraphy
. Angiography
A fifty-eight year o

10) A fifty-eight year old female patient presents to the emergency department in Calmette hospital
with a 24-hour history of several hematochesia. Physical examination show: hypotension, and
anemia. Both the upper endoscopy and colonoscopy are not diagnostic. The patient continues to pass
clots per rectum. Resuscitation has normalized her vital signs and maintained her Hct at 32%. What
is the most effective management strategy?
. Abdominal CT scan with contrast
. Magnetic resonance imaging
. Scintigraphy and angiography
. Emergency surgery with intraoperative enteroscopy
. Barium enema
A fifty-eight year o

11) 65-year-old woman has a prior history of hospitalization for UGI bleeding from a duodenal
ulcer. Which one of the following therapies is not useful for preventing recurrent ulcer hemorrhage?
. long-term maintenance therapy with full dose H2RA or daily PPI
. H. pylori eradication
. discontinuation of NSAID intake
. ulcer surgery
. bland diet
65-year-old woman ha

2
12) A 68-year-old healthy woman with a history of duodenal ulcer is placed on low-dose aspirin for
coronary prophylaxis. Two weeks later she presents to the emergency room in Calmette hospital
with one episode of melena . Physical examination showed : blood pressure and resting heart rate of
90 beats per minute without orthostatic changes. Melena is confirmed on rectal examination. Her
admission hematocrit is 36%. She is placed on highdose proton pump inhibitor therapy (omeprazole
40 mg b.i.d.). Due to other complications, endoscopy is not performed until the tenth hospital day,
and it shows a small (5 mm) duodenal ulcer with a clean base. Biopsy for CLO test is negative.
Which one of the following should be performed now?
. treat with misoprostol for an NSAID-related ulcer
. treat empirically with antibiotics for H. pylori
. obtain serum gastrin level to exclude Zollinger-Ellison syndrome
. perform another test to exclude H. pylori
. continue high-dose proton pump inhibitor therapy
A 68-year-old health

13) An 82-year-old debilitated woman with coronary artery disease and breast cancer presents with
hematochezia requiring 4 units of blood. Following colonic preparation, colonoscopy demonstrates
fresh blood in the left colon with marked diverticulosis. The right colon is normal and bilious
material is seen in the cecal pole. Upon withdrawal of the colonoscope, there was active oozing of
blood from the neck of a diverticulum in the distal sigmoid colon. The most appropriate management
now is
. supportive care with transfusion requirements as necessary
. technetium RBC scan
. angiography
. endoscopic therapy of the bleeding diverticulum
. immediate surgical therapy
An 82-year-old debil

14) A 65-year-old presents with iron-deficiency anemia, refractory to iron treatment. No history of
overt gastrointestinal bleeding, but fecal occult blood tests were positive on 4 of 5 occasions. A
previous EGD showed a large hiatal hernia. Two colonoscopies and a small bowel radiographic
series were unremarkable. An endoscopic finding of a repeat EGD is shown ulcerative esophagitis.
Which of the following is the most appropriate management of this condition?
. Endoscopic ligation
. Endoscopic sclerotherapy
. Laparoscopic Nissen fundoplication
. Distal esophagectomy
. Oral administration of a proton pump inhibitor twice daily and iron replacement therapy.
A 65-year-old presen

15) A 72-year-old woman with cryptogenic cirrhosis has developed progressive dyspnea for the past
3 months. Her laboratory investigations show hemoglobin of 8.4 g/dL with mean corpuscular
volume of 62 μm3 and serum ferritin level of 10 ng/mL. Stool specimens are brown but positive for
occult blood. Colonoscopy is normal. An endoscopic view of the gastric antrum is shown
Angiodysplasia. Which of the following is the most appropriate next step in management of this
condition?
. Partial gastrectomy or antrectomy
. Argon plasma coagulation
. Transjugular intrahepatic portosystemic shunts (TIPS)
. Oral administration of proton pump inhibitors twice daily
. Oral administration of non-selective beta-blockers
A 72-year-old woman

3
16) A 62-year-old woman with a past medical history of chronic renal insufficiency presents with
iron-deficiency anemia and intermittent melena for 2 months requiring multiple blood transfusions.
She underwent multiple endoscopic investigations, including EGD, colonoscopy, CT scan of the
abdomen and pelvis, and upper gastrointestinal series. No bleeding source was identified. Wireless
capsule endoscopy shows multiple vascular abnormalities in the jejunum . Which of the following is
the most like diagnosis?
. Dieulafoy’s lesions
. Anemia from chronic renal failure
. Vascular ectasia
. Crohn’s disease
. Small bowel diverticulosis
A 62-year-old woman

17) A 68-year-old man presents with intermittent episodes of hematochezia for several months. His
stools are normal, but he passes blood clots a few times per week. His past medical history is
significant for prostate cancer, for which he received radiation therapy 4 years ago. He denied any
weight loss or diarrhea. Physical examination is unremarkable. An endoscopic view of the rectum
showed: rectum appears with friability and telangiectasia pale. These findings are limited to the
distal rectum. Which of the following is the most likely diagnosis?
. Hereditary hemorrhagic telangiectasia
. Ulcerative proctitis
. Ischemic colitis
. Crohn’s colitis
. Radiation proctopathy
A 68-year-old man pr

18) Regarding patients with upper GI bleeds which is incorrect?


. Use of NSAIDs doubles the risk for an upper GI bleed
. Urea will increase relative to creatinine in acute bleed
. Presence of fresh blood on aspiration of NG tube increases mortality as opposed to a clear
aspirate.
. Active bleeding seen at endoscopy has a 10% risk of rebleeding after treatment.
. Most deaths occur from decompensation of other organ systems rather than
exsanguination.
Regarding patients w

19) Which of the following therapies is proven to reduce mortality and morbidity in bleeding peptic
ulcers?
. Endoscopic procedures
. H2 antagonists
. Proton pump inhibitors
. Octreotide
. Antacids
Which of the followi

20) Which is not true of bleeding oesophageal varices?


. Mortality approaches 25-40%
. Use of octreotide IV is as effective as sclerotherapy controlling bleeding in 74-92% of
cases.
. Sclerotherapy has a high rate of complications (40%) including perforation, aspiration,
pyrexia, chest pain, ulcers and strictures.

4
. Endoscopic variceal ligation is as effective as sclerotherapy with fewer side effects.
. Use of the sengstaken-blackmore tube is effective in controlling severe bleeding, is easy to
insert and has almost no complications.
Which is not true of

5
1) Chez un adulte suspect de pneumonie aiguë communautaire (PAC). Parmi les propositions
suivantes concernant les arguments cliniques de diagnostic de PAC, indiquez celle qui est vraie ?
Une température à 38,5 C (hyperthermie) permet d’affirmer le diagnostic
Une température à 37 C(apyrexie) permet d’éliminer le diagnostic
La présence de râles crépitants apparus récemment a une bonne valeur prédictive positive
L’absence de signe de gravités apparus récemment a une bonne valeur prédictive positive
Un taux normal de procalcitonine au 3ème jour a une bonne valeur prédictive positive
Chez un adulte suspe

2) Parmi les circonstances suivantes, quelle est celle qui peut rendre le diagnostic de pneumonie aiguë communautaire le
plus difficile ?
Cancer prostatique
Grossesse à terme
Fibrose hépatique
Insuffisance rénale
Patient âgé
Parmi les circonstan

3) Un malade chez lequel une pneumonie aiguë communautaire est diagnostiquée aux urgences doit être proposé au
réanimateur pour transfert si le critère est suivant est présent :
PAs < 170 mmHg
PaO2/FiO2 < 250

pH artériel < 7,42

Hb 10 g/dl

Epanchement minime

Un malade chez leque

4) Vous voyez en consultation à l’hôpital un homme de 52 ans, chauffeur de Tuk Tuk vivant seul dans la banlieue de

Phnom Penh. Il présente depuis 2 jours une toux et une sensation fébrile, non améliorés par l’achat d’un sachet de

médicaments à la pharmacie du quartier. Il va de plus en plus mal, et n’a plus la force de travailler.

A l’examen, vous notez : fréquence respiratoire à 31/min; fréquence cardiaque à 128/mn, pression artérielle à 110/80 mm

Hg. Température 37,7°C (sous sachets). Trouble de conscience; foyer de râles crépitants des lobes inférieurs droit et

gauche.

Quel critère clinique de gravité vous manque-t’il pour calculer le score CURB-65 ?

L’âge du patient

L’existence d’un coma

La socio-économie

La baisse de TA

Le taux d’urée plasmatique

Vous voyez en consul

1
5) Un sujet âgé de 36 ans consulte aux urgences pour une toux fébrile suspecte de pneumonie aiguë communautaire

(PAC). Parmi les propositions suivantes concernant les examens paracliniques d’imagerie pour la stratégie diagnostique

de PAC, indiquez celle qui est exacte ?

Une radiographie thoracique de face réalisée en 1ère intention

Une radiographie thoracique de face et de profil réalisée en 1ère intention

Une échographie pleuro-pulmonaire réalisée en 1ère intention

Une TDM thoracique avec IV de produits de contraste réalisée en 1ère intention


intention
Une TDM thoracique sans IV de produit de contraste réalisée en 1ère
Un sujet âgé de 36 a

6) Vous voyez en consultation une patiente de 43 ans, obèse, institutrice en école primaire à Takéo. Depuis 4 jours, elle

présente une rhinite, pharyngite, et une toux avec expectoration blanchâtre et mousseuse, associée à une douleur rétro-

sternale à l’inspiration. Elle a présenté une température pendant 2 jours, au maximum à 38,2°C, puis est devenue

apyrétique sans traitement. Mais elle consulte car la toux s’est majorée. A l’examen, T 37,2°C ; PA 120/70 mmHg ; FC

80/min ; FR 16/min ; SpO2 99% en air ambiant. L’auscultation pulmonaire retrouve quelques râles bronchiques, pas de

crépitants. Vous évoquez une bronchite aigue en 1èrehypothèse diagnostique, en pensant à une pneumopathie débutante

comme hypothèse de diagnostic différentiel.

Parmi les options de bilans paracliniques suivantes, laquelle choisissez-vous ?

Pas de prescription d’examens paracliniques

Prescription d’une radiographie thoracique de face

Prescription d’un bilan biologique : CRP et NFP

Prescription d’un bilan microbiologique : hémocultures et ECBC

Prescription d’un bilan virologique : écouvillon naso-pharyngé H5N1

Vous voyez en consul

7) vous recevez une femme XX âgée de 57 ans, pour une dyspnée fébrile. Elle est célibataire sans enfants et vit seule à

Phnom Penh. Elle est dyspnéique, se plaint d’une toux non productive et de douleurs thoraciques bi basales apparues en

même temps que la fièvre qui était d’emblée à 39,9°C depuis 24 heures. Elle n’a pas de maladies connues Les

2
prélèvements du sang et le cliché du thorax fait. Son élève qui l’emmène à l’hôpital dit qu’elle a une diarrhée

sanguinolente et un fébrile. Elle a prise le médicament anti diarrhéique, et il vous montre un sachet d’imodium. A

l’examen : fréquence respiratoire à 32/min, TA à 90/50 mmHg, FC à 137 bpm, T à 39°C, SpO2à 87% en air ambiant. Le

score de Glasgow est à 13, perte de conscience pendant le transport, râles crépitants bi basales. Dentition en mauvais état.

Le reste de l’examen est sans particularité. ECG : rythme régulier et sinusal, tachycardie à 128.Biologie : créatininémie

80 μmol/L; Na 138 mEq/L; Cl 100 mEq/L; K 3.1 mEq/L; glucose 0,8g/L; CRP 125 mg/L; GB 12 800 103/ml; PNN :

9500 103/ml; D-Dimères 500 ng/ml.

Donnez la bonne réponse concernant les principaux critères pour proposer une hospitalisation à cette patiente selon le

score Pneumonia Severity Index.

Son âge

Le score de Glasgow à 13

L’isolement social

Le risque d’inobservance

Le sexe féminin

vous recevez une fem

8) Une gazométrie artérielle est effectuée en air ambiant et retrouve : pH 7,47 ; Pa O2 60 mmHg ; PaCO2 32 mmHg ;

HCO323 mmHg. Sur la base de ces gaz du sang vous concluez que la patiente est en :

Acidose respiratoire

Alcalose respiratoire

Acidose métabolique

Alcalose métabolique

Hypercapnique

Une gazométrie artér

9) Dans le contexte, devant ces gaz du sang, on peut penser que

3
l’hypoxémie lié à une hypoventilation

l’hypercapnie liée à une hyperventilation

le shunt lié à une embolie pulmonaire

le shunt lié à une pneumopathie

le gaz du sang est de l’état de base

Dans le contexte, de

10) Vous demandez certains examens à visée bactériologique dans ce contexte. Indiquez la proposition exacte :

Ces examens ne doivent pas retarder l’antibiothérapie

L’antibiothérapie est débutée avant tout examen

Il faut attendre les résultats des examens pour débuter une antibiothérapie

Aucun examen bactériologique n’est réalisable car la toux de la patiente n’est pas productive

Aucun examen bactériologique n’est nécessaire si la patiente n’est pas hospitalisée en réanimation

Vous demandez certai

11) Votre externe vous demande si il y a un intérêt à réaliser une antigénurie pneumocoque.

Quels arguments lui répondez-vous ?

L’examen interprété sans difficile compte tenu de l’antécédent de sinusite

L’examen n’a aucun intérêt les symptômes ayant débuté moins de 24h avant

L’examen doit être réalisé avant le début de l’antibiothérapie

L’examen peut être réalisé après le début de l’antibiothérapie

L’examen a une sensibilité de 100%

Votre externe vous d

12) Vous allez admette la patienta. Quel est le score CURB-65 pour cette patiente ?

le score CURB-65 est de 0

4
le score CURB-65 est de 1

le score CURB-65 est de 2

le score CURB-65 est de 3

le score CURB-65 est de 4

Vous allez admette l

13) Vous hospitalisez le patient. Quelle est le traitement antibiotique vous prescrivez ?

Cotrimoxazole 960 mg PO, 3 fois par jour en 10 jours

Erythromycine 2g PO, 2 fois par jour en 10 jours

Amoxicilline 3g Injection repartir en 3 fois par jour puis relais PO en 10 jours

Ceftriaxone 2g une fois PO par jours en 10 jours

Lévofloxacine 750 mg PO, jour en 10 jours

Vous hospitalisez le

14) Elle quitte l’hôpital contre avis médical et revient 8 jours plus tard beaucoup plus calme, avec une douleur thoracique

qui est réapparue, une dyspnée et une hyperthermie à 38°C. Le cliché de thorax et les planches de scanner les plus

représentatives sont les suivantes.

Quelle est votre hypothèse diagnostique sur l’imagerie ?

Atélectasie pulmonaire

Excavation pulmonaire

Infiltration nodulaire

Opacité alvéolaire

Pleurésie cloisonnée

Elle quitte l’hôpita

15) Quels sont les principes de prise en charge que vous allez proposer pour vous come le 1èreannée internat?

5
Diurétique de l’angle avec Lasilix 40 mg 2 ampoules/jour

Drainage pleural avec prélèvement bactériologique

Kinésie respiratoire de drainage bronchique

Ponction pleurale avec évacuation

Décision débuter anti tuberculeux

Quels sont les princ

16) Un jeune home, auparavant sain, est admis aux urgences pour une intoxication aux barbituriques responsables d’une

hypoventilation sévère. La respiration mélange à 50% d’O2n’a pas modifié sa PaCO2. Approximative, à quelle valeur la

PaO2 peut être s’élever?

25

50

75

100

200

Un jeune home, aupar

17) On constate chez un patient ayant une inégalité du rapport ventilation/perfusion sévère et qui reçoit un mélange à

80% d’O2 n’a atteint que 300 mmHg. Les facteurs susceptible d’expliquer un chiffre si bas sont existence d’unités

pulmonaires à très bas VA /Q une atélectasie des unités à bas rapport VA/Q du a Les avantages des sondes nasales pour

l’administration d’O2 sont :

elles sont supportés confortablement sur le long période

les concentration d’O2 inspire dépasse facilement 60%

la concentration d’O2 dans l’air inspirée est connue avec précision

la PCO2 inspirée ne s’élève pas

On constate chez un

6
18) Les masques à oxygène avec effect Venturi sont utile parce que :

ils délivrent une concentration d’O2 superieur 60%

la concentration d’O2 inspirée est bien controlée

la PaCO2 inspirée est base

FiO2 s’élève lorsque la ventilation augmente

Les masques à oxygèn

19) L’oxygénothérapie intermittente peut être dangereuse dans le traitement de l’insuffisance respiratoire parce que:

l’élévation de la PaCO2 est rapide après arrêt de l’O2

la PaO2 s’effondre après arrêt de l’O2

l’élévation de la pression artérielle pulmonaire est importante

une HTA est possible

L’oxygénothérapie in

20) Chez quel(s) patients la prescription d’O2 est-elle potentiellement iatrogène ?

Chez tous les BPCO

Chez tous les Insuffisants respiratoires chroniques obstructifs

Chez tous les insuffisants respiratoires chroniques restrictifs

Chez tous les patients hypoxiques chroniques

Chez tous les patients hyperocéan

Chez quel(s) patient

Vous aimerez peut-être aussi